Расширенная матрица метод гаусса: Метод Гаусса онлайн

Содержание

принцип, теорема и примеры решения задач

Задание. Решить СЛАУ $\left\{\begin{array}{l} 2 x_{1}+x_{2}+x_{3}=2 \\ x_{1}-x_{2}=-2 \\ 3 x_{1}-x_{2}+2 x_{3}=2 \end{array}\right.$ методом Гаусса.

Решение. Выпишем расширенную матрицу системы и при помощи элементарных преобразований над ее строками приведем эту матрицу к ступенчатому виду (прямой ход) и далее выполним обратный ход метода Гаусса (сделаем нули выше главной диагонали). Вначале поменяем первую и вторую строку, чтобы элемент $a_{11}$ равнялся 1 (это мы делаем для упрощения вычислений):

$$\tilde{A}=A \mid B=\left(\begin{array}{rrr|r} 2 & 1 & 1 & 2 \\ 1 & -1 & 0 & -2 \\ 3 & -1 & 2 & 2 \end{array}\right) \sim\left(\begin{array}{rrr|r} 1 & -1 & 0 & -2 \\ 2 & 1 & 1 & 2 \\ 3 & -1 & 2 & 2 \end{array}\right)$$

Далее делаем нули под главной диагональю в первом столбце. Для этого от второй строки отнимаем две первых, от третьей — три первых:

$$\tilde{A} \sim\left(\begin{array}{rrr|r} 1 & -1 & 0 & -2 \\ 0 & 3 & 1 & 6 \\ 0 & 2 & 2 & 8 \end{array}\right)$$

Все элементы третьей строки делим на два (или, что тоже самое, умножаем на $\frac{1}{2}$ ):

$$\tilde{A} \sim\left(\begin{array}{rrr|r} 1 & -1 & 0 & -2 \\ 0 & 3 & 1 & 6 \\ 0 & 1 & 1 & 4 \end{array}\right)$$

Далее делаем нули во втором столбце под главной диагональю, для удобства вычислений поменяем местами вторую и третью строки, чтобы диагональный элемент равнялся 1:

$$\tilde{A} \sim\left(\begin{array}{ccc|c} 1 & -1 & 0 & -2 \\ 0 & 1 & 1 & 4 \\ 0 & 3 & 1 & 6 \end{array}\right)$$

От третьей строки отнимаем вторую, умноженную на 3:

$$\tilde{A} \sim\left(\begin{array}{rrr|r} 1 & -1 & 0 & -2 \\ 0 & 1 & 1 & 4 \\ 0 & 0 & -2 & -6 \end{array}\right)$$

Умножив третью строку на $\left(-\frac{1}{2}\right)$ , получаем:

$$\tilde{A} \sim\left(\begin{array}{rrr|r} 1 & -1 & 0 & -2 \\ 0 & 1 & 1 & 4 \\ 0 & 0 & 1 & 3 \end{array}\right)$$

Проведем теперь обратный ход метода Гаусса (метод Гассу-Жордана), то есть сделаем нули над главной диагональю. Начнем с элементов третьего столбца. Надо обнулить элемент $a_{23}$, для этого от второй строки отнимем третью:

$$\tilde{A} \sim\left(\begin{array}{rrr|r} 1 & -1 & 0 & -2 \\ 0 & 1 & 0 & 1 \\ 0 & 0 & 1 & 3 \end{array}\right)$$

Далее обнуляем недиагональные элементы второго столбца, к первой строке прибавляем вторую:

$$\tilde{A} \sim\left(\begin{array}{ccc|c} 1 & 0 & 0 & -1 \\ 0 & 1 & 0 & 1 \\ 0 & 0 & 1 & 3 \end{array}\right)$$

Полученной матрице соответствует система

$\left\{\begin{array}{l}x_{1}+0 \cdot x_{2}+0 \cdot x_{3}=-1 \\ 0 \cdot x_{1}+x_{2}+0 \cdot x_{3}=1 \\ 0 \cdot x_{1}+0 \cdot x_{2}+x_{3}=3\end{array}\right.$    или   $\left\{\begin{array}{l} x_{1}=-1 \\ x_{2}=1 \\ x_{3}=3 \end{array}\right.$

Ответ. $\left\{\begin{array}{l} x_{1}=-1 \\ x_{2}=1 \\ x_{3}=3 \end{array}\right.$

Системы линейных уравнений. Метод Гаусса

Рассмотрим систему линейных уравнений:

   

С этой системой связываются две матрицы: матрица коэффициентов

   

и расширенная матрица — с присоединенными свободными членами:

   

Элементарными преобразованиями системы линейных уравнений называются:

1. умножение уравнения на отличное от нуля число;

2. прибавление к одному уравнению любого другого, умноженного на любое число;

3. перестановка уравнений местами.

Теорема. Любая система линейных уравнений с помощью элементарных преобразований и, может быть, изменением нумерации неизвестных, может быть приведена к системе с трапециевидной матрицей.

Доказательство. Проводим элементарные преобразования только над строками матрицы , как в доказательстве теоремы о ранге матрицы. Возможно, при этом придется изменить нумерацию неизвестных. Приводим систему уравнений к виду

   

Если хотя бы одно из чисел отлично от нуля, то данная система уравнений решений не имеет (несовместна). Если же все они равны нулю, то последние равенств не несут никакой информации и могут быть отброшены. Тогда, если , то неизвестным можно придавать произвольные значения, а неизвестные находим из решения системы с треугольной матрицей

   

Эту систему удобно решать, определив из -го уравнения , затем из -го и т.д. Таким образом, можно выразить переменные через и получить общее решение системы. Если , то система (в случае совместности) имеет единственное решение.

Преобразование системы уравнений к системе с трапециевидной матрицей называется

прямым ходом метода Гаусса. Последовательное вычисление неизвестных в порядке называется обратным ходом.

Пример. Решить систему линейных уравнений

   

Решение. Составим расширенную матрицу системы:

   

Первую строку умножим на 3 и вычтем из второй. Затем первую строку умножим на 2 и вычтем из третьей. Получим

   

Далее вторую строку прибавим к третьей и отбросим нулевую строку, получим

   

Запишем полученные уравнения:

   

Из второго уравнения выразим :

   

Полученное выражение подставляем в первое уравнение и выражаем из него :

   

Ответ. Общее решение данной системы:

   

Задачи.

1. Решите систему линейных уравнений

   

2. Решите систему линейных уравнений

   

3. Решите систему линейных уравнений

   

в чем суть, решение системы уравнений, примеры с объяснением

Благодаря великим ученым было открыто множество эффективных теорем для работы со сложными математическими задачами. Один из таких примеров — метод Гаусса.

Метод Гаусса — что это такое

Метод Гаусса представляет собой методику эквивалентного преобразования исходной системы линейных уравнений в систему, решаемую существенно проще, чем исходный вариант.

Метод Гаусса используют для решения систем линейных алгебраических формул. Такой способ обладает рядом важных преимуществ:

  1. Нет необходимости сравнивать уравнения для оценки совместимости.
  2. Решение систем равенств, в которых число определителей совпадает или не совпадает с количеством неизвестных переменных.
  3. Поиск решений для уравнений с нулевым определителем.
  4. Сравнительно небольшое количество вычислительных операций для получения результата.

Основные определения и обозначения

Матрицы: определение и свойства

Такие системы являются наиболее удобным способом представления данных, с которыми впоследствии производят манипуляции. Матрица имеет вид прямоугольника для удобства расчетов. При использовании метода Гаусса работа осуществляется с треугольными матрицами, при записи которых применяется прямоугольник с нулями на тех местах, где числа отсутствуют. Часто нули не записывают, а только подразумевают.

Важным параметром матрицы является размер:

  • ширина — это количество строк, обозначают буквой m;
  • длину выражают числом столбцов, записывают буквой n.
Источник: bigpicture.ru

Размер матрицы будет записан в формате А m*n. В случае, когда m=n, матрица является квадратной, а m=n служит ее порядком. Номера строк и столбцов изменяются.

Определитель

Матрица обладает крайне важной характеристикой. Таким параметром является определитель. Данную величину рассчитывают с помощью диагонали. Для этого в матрице необходимо провести воображаемые диагональные линии. Затем следует найти произведение элементов, которые располагаются на этих диагоналях, а полученные значения суммировать таким образом:

  1. Если диагональ обладает наклоном в правую сторону, то знак «+».
  2. Для диагоналей, наклоненных влево, знак «–».
Источник: wp.com

Рассчитать определитель представляется возможным лишь в случае работы с квадратной матрицей.

Если необходимо определить данный параметр для прямоугольной матрицы, то следует выполнить следующие манипуляции:

  • из числа строк и числа столбцов выбрать наименьшее и обозначить его k;
  • отметить в матрице произвольным образом k столбцов и k строк.

Элементы, которые расположены на пересечении отмеченных столбцов и строк, образуют новую квадратную матрицу. В случае, когда определитель является числом, не равным нулю, то данный параметр будет обозначен как базисный минор первоначальной прямоугольной матрицы. Перед решением систем уравнений методом Гаусса полезно рассчитать определитель. Если данная характеристика равна нулю, то матрица имеет бесконечное множество решений либо не имеет их вовсе. В таком случае потребуется определить ранг матрицы.

Классификация систем

Ранг матрицы является распространенным понятием. Он обозначает максимальный порядок ее определителя, который не равен нулю. По-другому можно сказать, что ранг матрицы представляет собой порядок базисного минора. Исходя из данного критерия, СЛАУ классифицируют на несколько типов. В совместных системах, которые состоят лишь из коэффициентов, ранг основной матрицы совпадает с рангом расширенной. Для подобных систем характерно одно или множество решений. По этой причине совместные системы подразделяют на следующие типы:

  • определенные, обладающие одним решением, в которых наблюдается равенство ранга матрицы и количество неизвестных;
  • неопределенные;
  • обладающие бесконечным числом решений с рангом матрицы, который меньше количества неизвестных.

В несовместных системах ранги, характеризующие основную и расширенную матрицы, отличаются. С помощью метода Гаусса в процессе решения можно прийти либо к однозначному доказательству несовместности системы, либо к решению общего вида для системы, обладающей бесконечным количеством решений.

Источник: asiaplustj.info

Основные правила и разрешаемые преобразования при использовании метода Гаусса

Перед тем, как решать систему, необходимо ее упростить. На данном этапе выполняют элементарные преобразования, которые не влияют на конечный результат. Определенные манипуляции справедливы лишь в случае матриц, исходниками которых являются СЛАУ. Список элементарных преобразований:

  1. Перестановка строк. При перемене записей в системе местами ее решение не меняется. Можно менять место строк в матрице, учитывая столбец со свободными членами.
  2. Произведение всех элементов строк и некоторого коэффициента. Сокращаются большие числа в матрице, и исключаются нули. При этом множество решений сохраняется без изменений, а дальнейшие манипуляции существенно упрощаются. Важным условием является отличие от нуля коэффициента.
  3. Удаление строк, которые содержат пропорциональные коэффициенты. Данное преобразование следует из предыдущего пункта. При условии, что две или более строк в матрице обладают пропорциональными коэффициентами, то при произведении или делении одной из строк на коэффициент пропорциональности получают две или более абсолютно одинаковые строки. В этом случае лишние строки исключают, оставляя только одну.
  4. Удаление нулевой строки. Бывают случаи, когда в процессе манипуляций с уравнениями возникает строка, все элементы которой, в том числе свободный член, равны нулю. Нулевую строку допустимо исключать из матрицы.
  5. Суммирование элементов одной строки с элементами другой, умноженными на некоторый коэффициент, в соответствующих столбцах. Данное преобразование имеет наиболее важное значение из всех перечисленных.

Особенности использования метода Гаусса для решения СЛАУ

На первом этапе система уравнений записывается в определенном виде. Пример выглядит следующим образом:

Источник: wp.com

Коэффициенты необходимо представить в виде таблицы. С правой стороны в отдельном столбце записаны свободные члены. Данный блок отделен для удобства решения. Матрицу со столбцом со свободными членами называют расширенной.

Источник: wp.com

Затем основная матрица с коэффициентами приводится к верхней треугольной форме. Данное действие является ключевым моментом при решении системы уравнений с помощью метода Гаусса. По итогам преобразований матрица должна приобрести такой вид, чтобы слева внизу находились одни нули:

Источник: wp.com

При записи новой матрицы в виде системы уравнений можно отметить, что последняя строка уже содержит значение одного из корней, которое в дальнейшем подставляется в уравнение выше для нахождения следующего корня и так далее. Подобное описание позволяет разобраться в методе Гаусса в общих чертах.

Обратный и прямой ход метода Гаусса

В первом случае необходимо представить запись расширенной матрицы системы. При выполнении обратного метода Гаусса далее в главную матрицу добавляют столбец со свободными членами.

Источник: wp.com

Суть такого способа заключается в выполнении элементарных преобразований, по итогам которых данная матрица приводится к ступенчатому или треугольному виду. В этом случае над или под главной диагональю матрицы располагаются только нули.

Источник: wp.com

Варианты дальнейших действий:

  • перемена строк матрицы местами, при наличии одинаковых или пропорциональных строк их можно исключить, кроме одной;
  • деление либо умножение строки на любое число, не равное нулю;
  • удаление нулевых строк;
  • добавление строки, умноженной на число, не равное нулю, к другой строке.

Имея преобразованную систему с одной неизвестной Xn, которая становится известной, можно выполнить поиск в обратном порядке остальных неизвестных с помощью подстановки известных х в уравнения системы, вплоть до первого. Данный способ называют обратным методом Гаусса.

Примеры решений с объяснением

Пример 1

Требуется решить с помощью метода Гаусса систему линейных уравнений, которая выглядит следующим образом:

Источник: wp.com

Решение

Необходимо записать расширенную матрицу:

Источник: wp.com

Затем нужно выполнить преобразования. В результате матрица должна приобрести треугольный вид. Для этого следует умножить первую строку на (3) и умножить вторую строку на (-1). В результате суммирования второй и первой строк получается следующее:

Источник: wp.com

Далее следует умножить третью строку на (-1). После добавления третьей строки ко второй получаем следующие преобразования:

Источник: wp.com

После этого необходимо умножить первую строку на (6) и вторую строку на (13). Далее следует добавить вторую строку к первой:

Источник: wp.com

После того, как система преобразована, остается вычислить неизвестные:

\(x_{3}=\frac{98}{49}=2\)

\(x_{2}=\frac{14-7x_{3}}{6}=\frac{14-7*2}{6}=0\)

\(x_{3}=\frac{-9+5x_{2}+6x_{3}}{3}=\frac{-9+5*0+6*2}{3}=1\)

Данный пример демонстрирует единственное решение системы.

Источник: supertics.com

Пример 2

Необходимо решить систему уравнений, которая выглядит следующим образом:

Источник: wp.com

Решение

Необходимо составить матрицу:

Источник: wp.com

Согласно методу Гаусса уравнение первой строки по итогам преобразований не меняется. Удобнее, когда левый верхний элемент матрицы обладает наименьшим значением. В таком случае первые элементы остальных строк после преобразований будут равны нулю. Таким образом, составленная матрица будет решаться проще, если на место первой строки поставить вторую:

вторая строка:

\(k = (-a_{21} /a_{11}) = (-3/1) = -3\)

\(a»_{21} = a_{21} + k×a_{11} = 3 + (-3)×1 = 0\)

\(a» _{22} = a_{22} + k×a _{12} = -1 + (-3)×2 = -7\)

\(a»_{ 23} = a_{23} + k×a_{13} = 1 + (-3)×4 = -11\)

b» 2 = b 2 + k×b 1 = 12 + (-3)×12 = -24

третья строка: 

\(k = (-a_{31} /a_{11}) = (-5/1) = -5\)

\(a»_{31} = a_{31} + k×a_{11} = 5 + (-5)×1 = 0\)

\(a»_{32} = a_{32} + k×a_{12} = 1 + (-5)×2 = -9\)

\( a»_{33} = a_{33} + k×a_{13} = 2 + (-5)×4 = -18\)

\( b»_3 = b_3 + k×b_1 = 3 + (-5)×12 = -57\)

Матрица с промежуточными результатами манипуляций будет иметь следующий вид:

Источник: wp.com

Благодаря некоторым операциям можно придать матрице наиболее удобный вид. К примеру, вторую строку можно избавить от всех «минусов» путем умножения каждого элемента на «-1». Можно заметить, что для третьей строки характерны все элементы, кратные трем. В этом случае строка сокращается с помощью произведения каждого элемента на «-1/3». Минус позволит удалить отрицательные значения.

Источник: wp.com

Далее следует приступить к манипуляциям со второй и третьей строками. Необходимо суммировать третью и вторую строки. Вторая строка при этом умножается на такой коэффициент, при котором элемент а 32 будет равен нулю.

\(k = (-a_{32} /a_{22}) = (-3/7) = -3/7\)

В случае, когда некоторые преобразования приводят в результате к получению не целого числа, следует оставить его в этом виде. Таким образом, вычисления будут более точными. Затем при получении ответов можно определиться с его дальнейшем округлением или переводом в другую форму записи.

\(a»_{32} = a_{32} + k×a_{22} = 3 + (-3/7)×7 = 3 + (-3) = 0\)

\(a»_{33} = a_{33} + k×a_{23} = 6 + (-3/7)×11 = -9/7\)

\(b»_3 = b_3 + k×b_2 = 19 + (-3/7)×24 = -61/7\)

Преобразованная матрица будет иметь следующий вид:

 

 

Матрица обладает ступенчатым видом. Дальнейшие преобразования с помощью метода Гаусса нецелесообразны. В этом случае можно удалить из третьей строки общий коэффициент «-1/7».

Источник: wp.com

Затем необходимо представить запись матрицы в виде системы уравнений для вычисления корней.

x + 2y + 4z = 12 (1)

7y + 11z = 24 (2)

Найти корни можно обратным методом Гаусса. Уравнение (3) содержит значение z:

y = (24 — 11×(61/9))/7 = -65/9

С помощью первого уравнения можно определить х:

x = (12 — 4z — 2y)/1 = 12 — 4×(61/9) — 2×(-65/9) = -6/9 = -2/3

Подобная система является совместной и определенной, для которого характерно единственное решение. Ответ будет следующим:

x 1 = -2/3, y = -65/9, z = 61/9.

Метод Гаусса предполагает последовательное исключение неизвестных. Методика справедлива в случае решения квадратных систем линейных алгебраических уравнений. Несмотря на простоту метода, многие студенты сталкиваются с некоторыми трудностями в процессе поиска правильного решения. Это связано с наличием знаков «+» и «-». Поэтому для решения СЛАУ требуется проявить внимательность. А получить квалифицированную помощь можно на ресурсе Феникс.Хелп.

Метода Гаусса: примеры решения СЛАУ: понятия, определения, примеры задач

Найти решение этого же примера методом Гаусса в матричной форме записи:

3×1+2×2+x3+x4=-2×1-x2+4×3-x4=-1-2×1-2×2-3×3+x4=9×1+5×2-x3+2×4=4

Как решать?

Расширенная матрица системы представлена в виде:

   x1    x2     x3 x432111-14-1-2-2-3115-12-2-194

Прямой ход метода Гаусса в данном случае предполагает приведение расширенной матрицы к трапецеидальному виду при помощи элементарных преобразований. Этот процесс очень поход на процесс исключения неизвестных переменных в координатном виде.

Преобразование матрицы начинается с превращения всех элементов нулевые. Для этого к элементам 2-ой, 3-ей и 4-ой строк прибавляем соответствующие элементы 1-ой строки, которые умножены на -a21a11=-13, -a31a11=—23=23 и на -а41а11=-13.

Дальнейшие преобразования происходит по такой схеме: все элементы во 2-ом столбце, начиная с 3-ей строки, становятся нулевыми. Такой процесс соответствует процессу исключения переменной  . Для того, чтобы выполнить этой действие, необходимо к элементам 3-ей и 4-ой строк прибавить соответствующие элементы 1-ой строки матрицы, которая умножена на -а32(1)а22(1)=-23-53=-25 и -а42(1)а22(1)=-133-53=135:

   x1    x2     x3 x43211|-20-53113-43|-130-23-7353|2330133-4353|143~

      x1                 x2                           x3                           x4~3211|-20-53113-43|-130-23+(-25)(-53)-73+(-25)11353+(-25)(-43)|233+(-25)(-13)0133+135(-53)-43+135×11353+135(-43)|143+135(-13)~

       x1    x2     x3       x4~3211|-20-53113-43|-1300-195115|39500415-95|195

Теперь исключаем переменную x3 из последнего уравнения — прибавляем к элементам последней строки матрицы соответствующие элементы последней строки, которая умножена на а43(2)а33(2)=-415-195=4119.

       x1    x2     x3       x43211|-20-53113-43|-1300-195115|39500415-95|195~

      x1    x2               x3                           x4~3211|-20-53113-43|-1300-195115|39500415+4119(-195)-95+4119×115|195+4119×395~

       x1    x2     x3       x4~3211|-20-53113-43|-1300-195115|3950005619|39219

Теперь применим обратных ход метода. В матричной форме записи такое преобразование матрицы, чтобы матрица, которая отмечена цветом на изображении:

   x1    x2     x3       x43211|-20-53113-43|-1300-195115|3950005619|39219

стала диагональной, т.е. приняла следующий вид:

   x1    x2     x3       x43000|а10-5300|а200-1950|а30005619|39219, где а1, а2, а3 — некоторые числа.

Такие преобразования выступают аналогом прямому ходу, только преобразования выполняются не от 1-ой строки уравнения, а от последней. Прибавляем к элементам 3-ей, 2-ой и 1-ой строк соответствующие элементы последней строки, которая умножена на

-1155619=-209280, на —435619=1942 и на -15619=1956.

   x1    x2     x3       x43211|-20-53113-43|-1300-195115|3950005619|39219~

      x1    x2      x3                   x4~3211+(-1956)5619|-2+(-1956)392190-53113-43+1942×5619|-13+1942×3921900-195115+(-209280)5619|395+(-209280)392190005619|39219~

       x1    x2     x3       x4~3210|-90-531130|900-1950|-3850005619|39219

Далее прибавляем к элементам 2-ой и 1-ой строк соответствующие элементы 3-ей строки, которые умножены на

-113-195=5557 и на -1-195=519.

 x1    x2     x3       x43210|-90-531130|900-1950|-3850005619|39219~

      x1    x2             x3                   x4~321+519(-195)0|-9+519(-385)0-53113+5557(-195)0|9+5557(-385)00-1950|-3850005619|39219~

       x1    x2     x3       x4~3210|-110-5300|5300-1950|-3850005619|39219

На последнем этапе прибавляем элементы 2-ой строки к соответствующим элементам 1-ой строки, которые умножены на -2-53=65.

 x1    x2     x3       x43210|-110-5300|5300-1950|-3850005619|39219~

      x1           x2            x3      x4~32+65(-53)00|-11+65×53)0-5300|5300-1950|-3850005619|39219~

       x1    x2     x3       x4~3000|-90-5300|5300-1950|-3850005619|39219

Полученная матрица соответствует системе уравнений

3×1=-9-53×2=53-195×3=-3855619×4=39219, откуда находим неизвестные переменные.

Ответ: x1=-3, x2=-1,x3=2,x4=7.​​​

Методические рекомендации к внеурочной самостоятельной работе «Решение систем уравнений методом Гаусса»

Самостоятельная работа №2

Решение систем уравнений методом Гаусса

Цель работы: овладеть методом Гаусса при решении систем линейных уравнений

Студент должен:

Знать:

  • символику и формы записи систем линейных уравнений

  • что такое совместная и несовместная система уравнений

  • методы решения СЛАУ(метод Гаусса)

Уметь:

КОНТРОЛЬНЫЕ ВОПРОСЫ:

  1. В каком случае система имеет единственное решение?

  2. В каком случае система имеет бесконечное множество решений?

  3. В чем достоинство метода Гаусса по сравнению с другими методами?

Форма выполнения задания: решение задач (письменно)

Время выполнения 45 мин

Основной теоретический материал

Метод Гаусса. Этот способ заключается в обнулении элементов основной расширенной матрицы системы уравнений, находящихся под главной диагональю.

Ме́тод Га́усса — классический метод решения системы линейных алгебраических уравнений (СЛАУ). Это метод последовательного исключения переменных, когда с помощью элементарных преобразований система уравнений приводится к равносильной системе треугольного вида, из которой последовательно, начиная с последних (по номеру), находятся все переменные системы

Алгоритм метода Гаусса

  1. На основании системы линейных уравнений составляем расширенную матрицу системы;

  2. Приводим матрицу к «треугольному» виду;

  3. Определяем ранги основной и расширенной матриц, и на основании этого делаем вывод о совместности системы и количестве допустимых решений;

  4. В случае, если система имеет единственное решение производим обратную подстановку и находим его, если система имеет множество решений: выражаем базисные переменные через переменные которые могут принимать произвольные значения;

Для приведения исходной расширенной матрицы к треугольному виду используем следующие два свойства определителей:

Свойство 1. Определитель не изменит свое значение, если ко всем элементам какой-либо строки (столбца) матрицы прибавить соответствующие элементы параллельной строки (столбца), умноженные на произвольное одно и то же число.

Свойство 2. При перестановке двух любых столбцов или строк матрицы ее определитель меняет знак на противоположный, а абсолютная величина определителя остается неизменной.

На первом этапе составляется расширенная матрица, состоящая из коэффициентов при неизвестных, и с помощью несложных математических преобразований она приводится к виду, когда диагональ, состоящая из единичек отсекает нули:

  

На втором этапе последовательно находятся все неизвестные, начиная с предпоследней.

Решение типовых заданий

Решить систему трех линейных уравнений методом Гаусса

Ответ: х=1, y=2, z=3.

Покажем, как методом Гаусса можно решить следующую систему:

Обнулим коэффициенты при во второй и третьей строчках. Для этого вычтем из них первую строчку, умноженную на и , соответственно:

Теперь обнулим коэффициент при в третьей строке, вычтя из неё вторую строку, умноженную на :

В результате мы привели исходную систему к треугольному виду, тем самым закончим первый этап алгоритма.

На втором этапе разрешим полученные уравнения в обратном порядке. Имеем:

из третьего;

из второго, подставив полученное

из первого, подставив полученные и .

Ответ: (2; 3; -1).

Решить самостоятельно системы линейных уравнений по вариантам: 10 вариантов

1

2

3

4

5

6

7

8

9

10

Оформить отчет

Требования к оформлению самостоятельной работы

Расчетные задания должны быть выполнены в рабочей тетради

По результатам решения тренажера выставляется оценка, которая учитывается при приеме дифференцированного зачета.

Шкала оценки образовательных достижений

Процент результативности

(правильных ответов)

Интернет ресурсы

http://math2.ru/education/sys_lin_eq/gauss.html

http://www.bestreferat.ru/referat-180751.html

Оценка уровня подготовки

Балл (оценка)

Вербальный аналог

90-100

5

отлично

80-89

4

хорошо

70-79

3

удовлетворительно

менее 70

2

неудовлетворительно

Метод Гаусса — примеры c решением, теоремы и формулы

Метод Гаусса – идеальный вариант для решения систем линейных алгебраических уравнений (далее СЛАУ). Благодаря методу Гаусса можно последовательно исключать неизвестные путём элементарных преобразований. Метод Гаусса – это классический метод решения СЛАУ, который и рассмотрен ниже.

Карл Фридрих Гаусс – немецкий математик, основатель одноименного метода решения СЛАУ

Карл Фридрих Гаусс – был известным великим математиком и его в своё время признали «королём математики». Хотя название «метод Гаусса» является общепринятым, Гаусс не является его автором: метод Гаусса был известен задолго до него. Первое его описание имеется в китайском трактате «Математика в девяти книгах», который составлен между II в. до н. э. и I в. н. э. и представляет собой компиляцию более ранних трудов, написанных примерно в X в. до н. э.

Метод Гаусса – последовательное исключение неизвестных. Этот метод используется для решения квадратных систем линейных алгебраических уравнений. Хотя уравнения при помощи метода Гаусса решаются легко, но всё же студенты часто не могут найти правильное решение, так как путаются в знаках (плюсы и минусы). Поэтому во время решения СЛАУ необходимо быть предельно внимательным и только тогда можно легко, быстро и правильно решить даже самое сложное уравнение.

У систем линейных алгебраических уравнений есть несколько преимуществ: уравнение не обязательно заранее на совместность; можно решать такие системы уравнений, в которых число уравнений не совпадает с количеством неизвестных переменных или определитель основной матрицы равняется нулю; есть возможность при помощи метода Гаусса приводить к результату при сравнительно небольшом количестве вычислительных операций.

Определения и обозначения

Как уже говорилось, метод Гаусса вызывает у студентов некоторые сложности. Однако, если выучить методику и алгоритм решения, сразу же приходит понимание в тонкостях решения.

Для начала систематизируем знания о системах линейных уравнений.

Обратите внимание!

СЛАУ в зависимости от её элементов может иметь:

  1. Одно решение;
  2. много решений;
  3. совсем не иметь решений.

В первых двух случаях СЛАУ называется совместимой, а в третьем случае – несовместима. Если система имеет одно решение, она называется определённой, а если решений больше одного, тогда система называется неопределённой.

Метод Крамера и матричный способ не подходят для решения уравнений, если система имеет бесконечное множество решений. Вот поэтому нам и нужен метод Гаусса, который поможет нам в любом случае найти правильное решение. К элементарным преобразованиям относятся:

  • перемена мест уравнений системы;
  • почленное умножение обеих частей на одно из уравнений на некоторое число, так, чтобы коэффициенты при первой переменной в двух уравнениях были противоположными числами;
  • сложение к обеим частям одного из уравнений определённых частей другого уравнения.

Итак, когда мы знаем основные правила и обозначения, можно приступать к решению.

Теперь рассмотрим, как решаются системы методом Гаусса на простом примере:

   

где а, в, с  – заданные коэффициенты, d – заданные свободные члены, x, y, z – неизвестные. Коэффициенты и свободные члены уравнения можно называть его элементами.

Если = = = , тогда система линейных алгебраических уравнений называется однородной, в другом случае – неоднородной.

Множественные числа , , называются решением СЛАУ, если при подстановке , , в СЛАУ получим числовые тождества.

Система, которую мы написали выше имеет координатную форму. Если её переделать в матричную форму, тогда система будет выглядеть так:

– это основная матрица СЛАУ.

– матрица столбец неизвестных переменных.

– матрица столбец свободных членов.

Если к основной матрице добавить в качестве – ого столбца матрицу-столбец свободных членов, тогда получится расширенная матрица систем линейных уравнений. Как правило, расширенная матрица обозначается буквой , а столбец свободных членов желательно отделить вертикальной линией от остальных столбцов. То есть, расширенная матрица выглядит так:

Если квадратная матрица равна нулю, она называется вырожденная, а если – матрица невырожденная.

Обратите внимание!

Если с системой уравнений:          

Произвести такие действия:

  • умножать обе части любого из уравнений на произвольное и отличное от нуля число ;
  • менять местами уравнения;
  • к обеим частям любого из уравнений прибавить определённые части другого уравнения, которые умножаются на произвольное число ,

тогда получается эквивалентная система, у которой такое же решение или нет решений совсем.

Теперь можно перейти непосредственно к методу Гаусса.

Простейшие преобразования элементов матрицы

Мы рассмотрели основные определения и уже понимаем, чем нам поможет метод Гаусса в решении системы. Теперь давайте рассмотрим простую систему уравнений. Для этого возьмём самое обычное уравнение, где и используем решение методом Гаусса:

Из уравнения запишем расширенную матрицу:

Из данной матрицы видно, по какому принципу она записана. Вертикальную черту не обязательно ставить, но просто так удобнее решать систему.

Определение

Матрица системы – это матрица, которая составляется исключительно с коэффициентами при неизвестных. Что касается расширенной матрицы системы, так, это такая матрица, в которой кроме коэффициентов записаны ещё и свободные члены. Любую из этих матриц называют просто матрицей.

На матрице, которая написана выше рассмотрим, какие существуют элементарные преобразования:

1. В матрице строки можно переставлять местами. Например, в нашей матрице спокойно можно переставить первую и вторую строки:

.

2. Если в матрице имеются (или появились) пропорциональные строки (одинаковые), тогда необходимо оставить всего лишь одну строку, а остальные убрать (удалить).

3. Если в ходе преобразований в матрице появилась строка, где находятся одни нули, тогда такую строку тоже нужно удалять.

4. Строку матрицы можно умножать (делить) на любое число, которое отличное от нуля. Такое действие желательно проделывать, так как в будущем проще преобразовывать матрицу.

5. Сейчас рассмотрим преобразование, которое больше всего вызывает затруднение у студентов. Для этого возьмём изначальную нашу матрицу:

Для удобства умножаем первую строку на (-3):

Теперь ко второй строке прибавляем первую строку, которую умножали на -3. Вот что у нас получается:

В итоге получилось такое преобразование:

Теперь для проверки можно разделить все коэффициенты первой строки на те же и вот что получается:

В матрице верхняя строка преобразовалась:

Первую строку делим на и преобразовалась нижняя строка:

И верхнюю строку поделили на то же самое число :

Как вы можете убедиться, в итоге строка, которую мы прибавляли ни капельки не изменилась, а вот вторая строка поменялась. ВСЕГДА меняется только та строка, к которой прибавляются коэффициенты.

Мы расписали в таких подробностях, чтобы было вам понятно, откуда какая цифра взялась. На практике, например, на контрольной или экзамене матрица так подробно не расписывается. Как правило, в задании решение матрицы оформляется так:

.

Обратите внимание!

Если в примере приведены десятичные дроби, метод Гаусса в этом случае также поможет решить систему линейных алгебраических уравнений. Однако, не стоит забывать, что следует избегать приближённых вычислений, так как ответ будет неверным. Лучше всего использовать десятичные дроби, а от них переходить к обыкновенным дробям.

Алгоритм решения методом Гаусса пошагово

После того, как мы рассмотрели простейшие преобразования, в которых на помощь пришёл метод Гаусса, можем вернуться к нашей системе, которую уже разложили по полочкам и пошагово распишем:

Шаг 1. Переписываем систему в виде матрицы

Записываем матрицу:

Шаг 2. Преобразовываем матрицу: вторую строку в первом столбце приводим к нулю

Как мы привели вторую строку в первом столбце к нулю описано выше. Напомним, что первую строку умножали на и вторую строку прибавили к первой , умноженной на .

Шаг 3. Приводим матрицу к ступенчатому виду

Теперь вторую строку можно поделить на 2 и получается:

Верхнюю строку делим на и приводим матрицу к ступенчатому виду:

Когда оформляют задание, так и отчёркивают простым карандашом для упрощения работы, а также обводят те числа, которые стоят на “ступеньках”. Хотя в учебниках и другой литературе нет такого понятия, как ступенчатый вид. Как правило, математики такой вид называют трапециевидным или треугольным.

Шаг 4. Записываем эквивалентную систему

После наших элементарных преобразований получилась эквивалентная система:

Шаг 5. Производим проверку (решение системы обратным путём)

Теперь систему нужно решить в обратном направлении, то есть обратным ходом, начиная с последней строки.:

находим : ,

,

.

После находим :

,

.

Тогда:

.

Как видим, уравнение решено правильно, так как ответы в системе совпадают.

Решение систем линейных уравнений методом Гаусса, в которых основная матрица невырожденная, а количество в ней неизвестных равняется количеству уравнений

Как мы уже упоминали, невырожденная матрица бывает тогда, когда . Разберём систему уравнений невырожденной матрицы, где уравнений по количеству столько же, сколько и неизвестных. Эту систему уравнений решим другим способом.

Дана система уравнений:

Для начала нужно решить первое уравнение системы относительно неизвестной переменной . Далее подставим полученное выражение сначала во второе уравнение, а затем в третье, чтобы исключить из них эту переменную.

Теперь переходим ко второму уравнению системы относительно и полученный результат подставим в третье уравнение.. Это нужно для того, чтобы исключить неизвестную переменную :

Из последнего, третьего уравнения мы видим, что . Из второго уравнения находим . И последнее, находим первое уравнение .

Итак, мы нашли все три неизвестных при помощи последовательного исключения. Такой процесс называют – прямой ход метода Гаусса. Когда последовательно находятся неизвестные переменные, начиная с последнего уравнения, называется обратным ходом метода Гаусса.

Когда выражается через и в первом уравнении, а затем подставляется полученное выражение во второе или третье уравнения, тогда, чтобы привести в к такому же результату, необходимо проделать такие действия:

  • берём второе уравнение и к его левой и правой частям прибавляем определённые части из первого уравнения, которые умножаются на ,
  • берём третье уравнение и к его левой и правой частям прибавляем определённые части из первого уравнения, которые умножаются на .

И действительно, благодаря такой процедуре у нас есть возможность исключать неизвестную переменную со второго и третьего уравнения системы:

Возникают нюансы с исключением неизвестных переменных тогда, когда в уравнении системы нет каких-либо неизвестных переменных. Рассмотрим такую систему:

В этой системе в первом уравнении нет переменной и поэтому у нас нет возможности решить первое уравнение системы относительно , чтобы исключить данную переменную из остальных уравнений. В таком случае выход есть. Нужно всего лишь уравнения переставить местами.

Так как мы описываем уравнения системы, в которых определитель основных матриц отличен от нуля, тогда всегда есть такое уравнение, в котором есть необходимая нам переменная и это уравнение мы можем поставить туда, куда нам нужно.

В примере, который мы рассматриваем, достаточно всего лишь поменять местами первое и второе уравнение.

Теперь мы можем спокойно разрешить первое уравнение относительно переменной и убрать (исключить) из остальных уравнений в системе. Вот и весь принцип работы с такими, на первый взгляд, сложными системами.

Решение систем линейных уравнений методом Гаусса, в которых основная матрица вырожденная, а количество в ней неизвестных не совпадает с количеством уравнений

Метод Гаусса помогает решать системы уравнений, у которых основная матрица прямоугольная или квадратная, но основная вырожденная матрица может совсем не иметь решений, иметь бесконечное множество решений или иметь всего лишь одно единственное решение.

Рассмотрим, как при помощи метода Гаусса устанавливается совместность или несовместность систем линейных уравнений. В случае, если есть совместность определим все решения или одно решение.

В принципе, исключать неизвестные переменные можно точно так, как описано выше. Однако, есть некоторые непонятные ситуации, которые могут возникнуть в ходе решения:

1. На некоторых этапах в момент исключения неизвестных переменных некоторые уравнения могут обратиться в тождества . В данном случае такие уравнения лишние в системе и их можно смело полностью убирать, а затем продолжать решать уравнение методом Гаусса.

Например, вам попалась подобная система:

У нас получается такая ситуация

Как видим, второе уравнение . Соответственно, данное уравнение мы можем из системы удалить, так как оно без надобности.

Дальше можно продолжать решение системы линейных алгебраических уравнений уравнений традиционным методом Гаусса.

2. При решении уравнений прямым ходом методом Гаусса могут принять не только одно, но и несколько уравнений такой вид: , где – число, которое отличное от нуля. Это говорит о том, что такое уравнение никогда не сможет превратиться в тождество даже при любых значениях неизвестных переменных. То есть, можно выразить по-другому. Если уравнение приняло  вид, значит система несовместна, то есть, не имеет решений. Рассмотрим на примере:

Для начала необходимо исключить неизвестную переменную из всех уравнений данной системы, начиная со второго уравнения. Для этого нужно прибавить к левой и правой частям второго, третьего, четвёртого уравнения части (левую и правую) первого уравнения, которые соответственно, умножаются на (-1), (-2), (-3). Получается:

В третьем уравнении получилось равенство . Оно не подходит ни для каких значений неизвестных переменных , и , и поэтому, у данной системы нет решений. То есть, говорится, что система не имеет решений.

3. Допустим, что при выполнении прямого хода методом Гаусса нам нужно исключить неизвестную переменную , и ранее, на каком-то этапе у нас уже исключалась вместе с переменной . Как вы поступите в таком случае? При таком положении нам нужно перейти к исключению переменной . Если же  уже исключались, тогда переходим к ,  и т. д.

Рассмотрим систему уравнений на таком этапе, когда уже исключилась переменная :

Такая система уравнений после преобразования выглядит так:

Вы наверное уже обратили внимание, что вместе с исключились и . Поэтому решение методом Гаусса продолжаем исключением переменной из всех уравнений системы, а начнём мы с третьего уравнения:

Чтобы завершить уравнение прямым ходом метода Гаусса, необходимо исключить последнюю неизвестную переменную из последнего уравнения:

Допусти, что система уравнений стала:

В этой системе нет ни одного уравнения, которое бы сводилось к . В данном случае можно было бы говорить о несовместности системы. Дальше непонятно, что же делать? Выход есть всегда. Для начала нужно выписать все неизвестные, которые стоят на первом месте в системе:

В нашем примере это , и . В левой части системы оставим только неизвестные, которые выделены зелёным квадратом а в правую перенесём известные числа, но с противоположным знаком. Посмотрите на примере, как это выглядит:

Можно придать неизвестным переменным с правой части уравнений свободные (произвольные) значения: , , , где , ,  – произвольные числа.

Теперь в правых частях уравнений нашей системы имеются числа и можно приступать к обратному ходу решения методом Гаусса.

В последнем уравнении системы получилось: , и теперь мы легко найдём решение в предпоследнем уравнении: , а из первого уравнения получаем:

= =

В итоге, получился результат, который можно и записать.

Ответ

,

,

,

,

,

.

Примеры решения методом Гаусса

Выше мы подробно расписали решение системы методом Гаусса. Чтобы закрепить материал, решим несколько примеров, в которых опять нам поможет метод Гаусса. Соответственно, начнём с самой простой системы.

Пример 1

Задача 

Решить систему линейных алгебраических уравнений методом Гаусса:

Решение

Выписываем матрицу, куда добавляем столбец свободных членов:

Прежде всего мы смотрим на элемент, который находится в матрице в левом верхнем углу (первая строка, первый столбец). Для наглядности выделим цифру зелёным квадратом. На этом месте практически всегда стоит единица:

Так как мы должны использовать подходящее элементарное преобразование строк и сделать так, чтобы элемент, который находится в матрице под выделенной цифрой превратился в . Для этого можно ко второй строке прибавить первую строку и умножить на .Однако, не сильно хочется работать с дробями, поэтому давайте постараемся этого избежать. Для этого нужно вторую строку умножить на (разрешающий элемент данного шага).

Соответственно, первая строка остаётся неизменной, а вторая поменяется:

Подбираем такое элементарное преобразование строк, чтобы во второй строке в первом столбце образовался . Для этого первую строку нужно умножить на и только после этого ко второй строке прибавить изменённую после умножения на вторую строку. Вот что получилось:

. Теперь прибавляем со второй строки первую строку . У нас получился , который записываем во вторую строку в первый столбец. Также решаем и остальные элементы матрицы. Вот что у нас получилось:

Как всегда у нас первая строка осталась без изменений, а вторая с новыми числами.

Итак, у нас получился ступенчатый вид матрицы:

Записываем новую систему уравнений:

Для проверки решаем систему обратным ходом. Для этого находим сначала :

Так как найден, находим :

.

Подставляем в изначальную нашу систему уравнений найденные и :

и .

Как видите из решения, система уравнений решена верно. Запишем ответ.

Ответ

Выше мы решали систему уравнений в двумя неизвестными, а теперь рассмотрим систему уравнений с тремя неизвестными.

Пример 2

Задача

Решить систему уравнений методом Гаусса:

Решение

Составляем матрицу, куда вписываем и свободные члены:

Что нам надо? Чтобы вместо цифры 2 появился 0. Для этого подбираем ближайшее число. Например, можно взять цифру -2 и на неё перемножить все элементы первой строки. Значит, умножаем , а потом прибавляем, при этом задействуем вторую строку: . В итоге у нас получился нуль, который записываем во вторую строку в первый столбец. Затем , и . Аналогично, и . И умножаем свободный член . Так и запишем следующую матрицу. Не забывайте, что первая строка остаётся без изменений:

Дальше необходимо проделать те же самые действия по отношению к третьей строке. То есть, первую строку нужно умножать не на (-2), а на цифру 3, так как и в третьей строке нужно коэффициенты привести у нулю. Также первую строку умножаем на 3 и прибавляем третью строку. Получается так:

Теперь нужно обнулить элемент 7, который стоит в третьей строке во втором столбце. Для этого выбираем цифру (-7) и проделываем те же действия. Однако, необходимо задействовать вторую строку. То есть, вторую строку умножаем на (-7) и прибавляем с третьей строкой. Итак, . Записываем результат в третью строку. Такие же действия проделываем и с остальными элементами. Получается новая матрица:

В результате получилась ступенчатая система уравнений:

Сначала находим : ,

.

Обратный ход:

Итак, уравнение системы решено верно.

Ответ

,

,

.

Пример 3

Система с четырьмя неизвестными более сложная, так как в ней легко запутаться. Попробуем решить такую систему уравнений.

Задача

Решите систему уравнений методом Гаусса:

Решение                                                                

В уравнении , то есть – ведущий член и пусть  ≠ 0

Из данного уравнения составим расширенную матрицу:

Теперь нужно умножить последние три строки (вторую, третью и четвёртую) на: , , . Затем прибавим полученный результат ко второй, третьей и четвёртой строкам исключаем переменную из каждой строки, начиная не с первой, а не со второй. Посмотрите, как изменилась наша новая матрица и в теперь стоит 0.

Поменяем вторую и третью строку местами и получим:

Получилось так, что = b и тогда, умножая вторую строку на (-7/4) и результат данной строки, прибавляя к четвёртой, можно исключить переменную из третьей и четвёртой строк:

Получилась такая матрица:

Также, учитывая, что  = , умножим третью строку на: 13,5/8 = 27/16, и, полученный результат прибавим к четвёртой, чтобы исключить переменную и получаем новую систему уравнений:

Теперь необходимо решить уравнение обратным ходом и найдём из последнего, четвёртого уравнения ,

из третьего: = = =

второе уравнение находим: = = = 2,

из первого уравнения: = .

Значит, решение системы такое: (1, 2, -1, -2).

Ответ

,

,

,

.

Добавим ещё несколько примеров для закрепления материла, но без такого подробного описания, как предыдущие системы уравнений.

Пример 4

Задача

Решить систему уравнений методом Гаусса:

Решение

Записываем расширенную матрицу системы:

Сначала смотрим на левое верхнее число:

Как выше уже было сказано, на этом месте должна стоять единица, но не обязательно. Производим такие действия: первую строку умножаем на -3, а потом ко второй строке прибавляем первую:

 

Производим следующие действия: первую строку умножаем на -1. Затем к третьей строки прибавляем вторую:

Теперь вторую строку умножаем на 1, а затем к третьей строке прибавляем вторую:

Получился ступенчатый вид уравнения:

Проверяем:

,

,

,

,

.

.

  Ответ

,

,

.

Заключение

Итак, вы видите, что метод Гаусса – интересный и простой способ решения систем линейных алгебраических уравнений. Путём элементарных преобразований нужно из системы исключать неизвестные переменные, чтобы систему превратить в ступенчатый вид. Данный метод удобен тем, что всегда можно проверить, правильно ли решено уравнение. Нужно просто подставить найденные неизвестные в изначальную систему уравнений.

Если элементы определителя не равняются нулю, тогда лучше обратиться к методу Крамера, а если же элементы нулевые, тогда такие системы очень удобно решать благодаря методу Гаусса.

Предлагаем ещё почитать учебники, в которых также описаны решения систем методом Гаусса.

Литература для общего развития:

Умнов А. Е. Аналитическая геометрия и линейная алгебра, изд. 3: учеб. пособие – М. МФТИ – 2011 – 259 с.

Карчевский Е. М. Лекции по линейной алгебре и аналитической геометрии, учеб. пособие – Казанский университет – 2012 – 302 с.

Метод Гаусса – теорема, примеры решений обновлено: 16 апреля, 2020 автором: Научные Статьи.Ру

Численные методы: решение систем линейных уравнений

В прикладных задачах часто возникает необходимость решать системы линейных уравнений.

Система линейных алгебраических уравнений с n неизвестными  —  это система уравнений вида

                                     (1)

Слово система означает, что все уравнения рассматриваются как одно целое.

В общем случае у нас имеется m — уравнений, n — количество неизвестных. x1x2,…, xn — неизвестные, которые следует определить.

В системе (1)  – фиксированные коэффициенты,  b1b2, …, bm — свободные члены — предполагаются известными.

Система (1) называется однородной, если все её свободные члены равны нулю (b1 = b2 = … = bm = 0), иначе — неоднородной.

Система (1) называется квадратной, если число m уравнений равно числу n неизвестных.

Задача состоит в том, чтобы найти такие  которые удовлетворяют всем уравнениям (1).

В частном случае мы имеем одно линейное уравнение:

Конечно, такое уравнение легко решить, если предположить, что коэффициент  не равен 0, имеем:  = .

Очевидно, в общем случае имеются 3 варианта решений: система имеет ни одного решения, имеет одно решение, более одного решения.

Система (1) называется совместной, если она имеет хотя бы одно решение, и несовместной, если нет ни одного решения.

Система линейных уравнений может быть представлена в матричной форме как:

или:

Ax = b

Здесь A — это матрица системы, x — столбец неизвестных, а b — столбец свободных членов.

Если к матрице A приписать справа столбец свободных членов, то получившаяся матрица называется расширенной.

Рассмотрим, например, систему вида и поймем, как найти ее решение:

                                      (2)

Предположим на минуту, что в первом уравнении y отсутствует, а во втором отсутствует x, тогда мы имели бы решение именно то решение, которое нам нужно.

Вопрос: как исходную систему привести к такому виду и можно ли это сделать.

Заметим, что с тождествами мы можем делать следующие вещи: домножать на одно и то же число, отличное от 0, складывать, вычитать и тд, это похоже с тем, что вы раскладываете монеты по своим карманам, не меняя общей суммы.

От этих операций тождество не меняется.

В системе (2) у нас два тождества, домножим второе тождество на 2 и вычтем из первого, получим:

                                      (3)

Формально у нас есть еще старое тождество , но оно нам не понадобится (подумайте, почему).

Система (3) точно такая же, как система (2).

Из второго уравнения системы (3) сразу получим:

 

Никто не мешает нам подставить это значение в первое уравнение:

Отсюда сразу находим, что

Итак, путем простых действий мы нашли, что система (2) может быть представлена в виде:

Именно такие естественные соображения приводят к общему методу решения систем линейных уравнений, известному как метод исключения или метод Гаусса.

Метод Гаусса является одним из самых распространенных прямых методов решения систем линейных уравнений Ax = b:

Опишем этот метод в общем случае.

Вначале исходная система приводится к верхнетреугольному виду.

Это достигается следующей последовательностью преобразований (прямой ход).

Будем считать для удобства, что элемент aij исходной матрицы и компоненты вектора bi есть, соответственно, элементы aij (1) первого шага преобразованной матрицы A1 и преобразованного вектора b1:A = A1, b=b1

Далее, на втором шаге прибавим к второй строке первую, умноженную на  

Аналогично поступим со всеми оставшимися строками, т.е. прибавим к каждой i-ой строке i=2,3,…,N, первую, умноженную на коэффициент  

При этом соответственно изменится и вектор b1. 

Таким образом, 2 шаг.

Имеем систему уравнений A2x = b2:

где

3 шаг.

Прибавим к новой третьей строке новую вторую, умноженную на  

То же самое сделаем с остальными строками 4,5,…,N, т.е. прибавим к i-ой строке вторую, умноженную на  

При этом получим систему A3x = b3:

(k+1)-ый шаг:

Здесь

Поступая так и далее, на шаге N-1 получаем верхнетреугольную систему:

При этом, мы также получили матрицу C переводных коэффициентов, имеющую вид:

Решение полученной треугольной системы  как легко видеть, имеет вид (обратный ход метода Гаусса):

Заметим, что при прямом ходе метода Гаусса может возникнуть ситуация, когда происходит деление на нуль, да и вообще, желательно не делить на малое число, чтобы не накапливалась ошибка.

Поэтому метод Гаусса обычно проводят с частичным выбором главного элемента, то есть после каждого шага (пусть это был k-й шаг) переставляют строки с номерами k,k+1,…,N таким образом, чтобы на месте kk оказался элемент  наибольший из всех в k-ом столбце при m>k (при этом, естественно, переставляются и компоненты вектора b).

Можно для максимальной точности переставлять также и столбцы преобразуемой матрицы, чтобы на месте kk оказался максимальный элемент из всех с индексами больше, либо равными k.

Эта процедура называется методом Гаусса с выбором главного элемента. Она несколько повышает точность по сравнению с частичным выбором главного элемента, но весьма неудобна, в том числе для программирования, поскольку при перестановке строк компоненты искомого вектора x переставлять не надо, тогда как при перестановке столбцов надо переставлять и соответствующие компоненты вектора x.

Опишем обратный ход метода Гаусса в несколько иной форме (треугольное разложение).

Введем матрицы Mk по правилу:

На каждом шаге метода Гаусса получается некоторая промежуточная матрица: 

 и вектор  

Нетрудно видеть, что

Вопрос. Почему

Если производить также выбор главных элементов, то необходимо использовать оператор P перестановки индексов l и m, матричные элементы которого равны:

При применении оператора перестановки индексов к матрице слева, меняются местами строки матрицы и компоненты свободного вектора (PAx = Pb), если же его применить справа к матрице, то меняются местами ее столбцы и компоненты решения

Существует большой класс так называемых итерационных методов решения систем уравнений, аналогичных итерационным методам нахождения корней нелинейных уравнений.

Итерационные методы последовательно уточняют решение, отправляясь от начального приближения.

При выполнении условий сходимости они позволяют достичь любой точности просто повторением итераций.

Преимущество этих методов в том, что часто они позволяют достичь решения с заранее заданной точностью быстрее, а также позволяют решать большие системы уравнений.

Идея состоит в том, чтобы найти неподвижную точку матричного уравнения

                                     (5)

эквивалентного начальной системе линейных алгебраических уравнений.

При итерации  в правой части уравнения заменяется, например, в методе Якоби (метод простой итерации) приближение, найденное на предыдущем шаге:

.

Термин неподвижная точка становится ясен, если вы внимательно посмотрите на уравнение (5), по самому своему смыслу величина Х является неподвижной точкой.

Более подробное описание методов решения систем линейных уравнений можно найти в специальной литературе, наша задача дать обзор методов и основные идеи решения такого рода задач.

Обусловленность линейных систем, погрешность

При решении абстрактной задачи Ax = b, где A — оператор произвольной природы, важным моментом является корректность ее постановки.

Задача считается корректной, если решение существует и единственно и , кроме того, решение непрерывно зависит от данных (то есть, при  также стремится к нулю).

Однако и непрерывная зависимость от входных данных может иметь свои нюансы.

Чем меньшее (большее) изменение решения вызывает вариация входных данных, тем более хорошо (плохо) обусловленной считается задача.

Понятие обусловленности является тем более существенным для численных методов, поскольку на практике входные данные известны, как правило, с некоторой погрешностью.

Кроме того, существуют ошибки округления, возникающие при вычислениях.

Таким образом, формально корректная задача, являясь плохо обусловленной, может оказаться разрешимой столь неточно, что в этом будет отсутствовать практический смысл.

Чем можно охарактеризовать количественно обусловленность для линейных систем?

Пусть A — квадратная NxN — матрица.

Рассмотрим задачу Ax = b.

Пусть также  некоторая норма в пространстве RN 

Норма оператора A определяется стандартно:

Обозначим y = Ax и введем число m по правилу:

Величина  называется числом обусловленности.

Очевидно:

  1.      
  2. если A — диагональная, то  (Для какой нормы, или для всех вышеприведенных?). Чем меньше число обусловленности C(A), тем лучше обусловлена система. Действительно, пусть  вариация правой части, а соответствующее изменение решения.

Тогда справедливо следующее неравенство:

 

Доказательство. Имеем:

Так как

то    

Аналогично, поскольку  

Объединяя два неравенства, окончательно получаем для оценки погрешности:

 

В начало

Содержание портала

Исключение Гаусса

Тип 2. Умножьте строку на ненулевую константу.

Тип 3. Добавьте одну строку, кратную одной, в другую.

Цель этих операций — преобразовать — или уменьшить — исходную расширенную матрицу в одну из форм, где A ′ является верхним треугольником ( a ij ′ = 0 для i> j ), любые нулевые строки появляются внизу матрицы, и первая ненулевая запись в любой строке находится справа от первой ненулевой записи в любой более высокой строке; такая матрица имеет вид эшелон .Решения системы представлены более простой расширенной матрицей [ A ′ | b ′], можно найти путем осмотра нижних рядов и обратной подстановки в более высокие ряды. Поскольку элементарные операции со строками не меняют решений системы, векторы x , которые удовлетворяют более простой системе A x = b ′, являются в точности теми, которые удовлетворяют исходной системе, A x = b .

Пример 3 : Решите следующую систему с помощью исключения Гаусса:

Расширенная матрица, которая представляет эту систему:

Первая цель — получить нули под первой записью в первом столбце , что означает исключение первой переменной x из второго и третьего уравнений.Для этого выполняются следующие операции со строками:

Вторая цель — получить ноль под второй записью во втором столбце, что означает исключение второй переменной y из третьего уравнения. Один из способов добиться этого — добавить -1/5 второй строки к третьей строке. Однако, чтобы избежать дробей, есть еще один вариант: сначала поменять местами второй и третий ряды. Замена двух строк просто меняет местами уравнения, что явно не изменит решения системы:

Теперь прибавьте −5 раз вторую строку к третьей строке:

Поскольку матрица коэффициентов преобразована в эшелонированную форму, «прямая» часть исключения Гаусса завершена.Теперь остается использовать третью строку для оценки третьего неизвестного, затем выполнить обратную подстановку во вторую строку для оценки второго неизвестного и, наконец, выполнить обратную замену в первой строке для оценки первого неизвестного.

Третья строка финальной матрицы переводится в 10 z = 10, что дает z = 1. Обратная подстановка этого значения во вторую строку, которая представляет уравнение y — 3 z = — 1, дает y = 2.Обратная подстановка обоих этих значений в первую строку, которая представляет уравнение x -2 y + z = 0, дает x = 3. Таким образом, решение этой системы: ( x, y, z ) = (3, 2, 1).

Пример 4 : Решите следующую систему с помощью исключения Гаусса:

Для этой системы расширенная матрица (вертикальная линия опущена) составляет

Сначала умножьте строку 1 на 1/2:

Теперь добавление -1 первой строки ко второй строке дает нули под первой записью в первом столбце:

Перестановка второй и третьей строк дает желаемую матрицу коэффициентов верхней треугольной формы:

В третьей строке теперь указано z = 4.Обратная подстановка этого значения во вторую строку дает y = 1, а обратная подстановка обоих этих значений в первую строку дает x = −2. Таким образом, решение этой системы ( x, y, z ) = (−2, 1, 4).

Исключение Гаусса-Джордана . Исключение Гаусса осуществляется путем выполнения элементарных операций со строками для получения нулей ниже диагонали матрицы коэффициентов, чтобы привести ее к эшелонированной форме. (Напомним, что матрица A ′ = [ a ij ′] имеет эшелонированную форму, когда a ij ′ = 0 для i> j , любые нулевые строки появляются в нижней части матрицы , и первая ненулевая запись в любой строке находится справа от первой ненулевой записи в любой более высокой строке.Как только это будет сделано, проверка нижней строки (строк) и обратная подстановка в верхние строки определяют значения неизвестных.

Однако можно сократить (или полностью исключить) вычисления, связанные с обратной подстановкой, выполнив дополнительные операции со строками для преобразования матрицы из эшелонированной формы в сокращенную форму . Матрица находится в форме сокращенного эшелона, когда, помимо того, что она находится в форме эшелона, каждый столбец, содержащий ненулевую запись (обычно равную 1), имеет нули не только под этой записью, но и над этой записью.Грубо говоря, гауссовское исключение работает сверху вниз, чтобы создать матрицу в форме эшелона, тогда как Гаусс-Жорданов исключение продолжается с того места, где остановился гауссиан, затем работает снизу вверх для создания матрицы в форме сокращенного эшелона. Техника будет проиллюстрирована на следующем примере.

Пример 5 : Известно, что высота, y , брошенного в воздух объекта задается квадратичной функцией от t (время) в форме y = at 2 + bt + c .Если объект находится на высоте y = 23/4 в момент времени t = 1/2, при y = 7 в момент времени t = 1 и при y = 2 при t = 2 , определите коэффициенты a, b и c .

Так как t = 1/2 дает y = 23/4

, а два других условия, y ( t = 1) = 7 и y ( t = 2) = 2, дают следующие уравнения для a, b и c :

Следовательно, цель — решить систему

Расширенная матрица для этой системы сокращается следующим образом:

На этом прямая часть исключения Гаусса завершена, поскольку матрица коэффициентов приведена к эшелонированной форме.Однако, чтобы проиллюстрировать исключение Гаусса-Жордана, выполняются следующие дополнительные элементарные операции со строками:

Эта окончательная матрица сразу дает решение: a = −5, b = 10 и c = 2.

Пример 6 : Решите следующую систему с помощью исключения Гаусса:

Расширенная матрица для этой системы —

Кратные значения первой строки добавляются к другим строкам, чтобы получить нули под первой записью в первом столбце:

Затем −1 раз вторая строка добавляется к третьей строке:

В третьей строке теперь указано 0 x + 0 y + 0 z = 1, уравнение, которому не могут удовлетворять никакие значения x, y и z .Процесс останавливается: у этой системы нет решений.

Предыдущий пример показывает, как метод исключения по Гауссу обнаруживает противоречивую систему. Небольшое изменение этой системы (например, изменение постоянного члена «7» в третьем уравнении на «6») проиллюстрирует систему с бесконечным числом решений.

Пример 7 : Решите следующую систему с помощью исключения Гаусса:

Те же операции, которые применяются к расширенной матрице системы в примере 6, применяются к расширенной матрице для данной системы:

Здесь третья строка переводится в 0 x + 0 y + 0 z = 0, уравнение, которому удовлетворяют любые x, y и z .Поскольку здесь нет ограничений на неизвестные, на неизвестные не три условия, а только два (представленные двумя ненулевыми строками в окончательной расширенной матрице). Поскольку имеется 3 неизвестных, но только 2 константы, 3–2 = 1 неизвестных, скажем, z , произвольно; это называется свободной переменной . Пусть z = t , где t — любое действительное число. Обратная подстановка z = t во вторую строку (- y + 5 z = −6) дает

Обратная подстановка z = t и y = 6 + 5 t в первую строку ( x + y -3 z = 4) определяет x :

Следовательно, каждое решение системы имеет вид

, где t — любое действительное число.Существует бесконечно много решений, поскольку каждое действительное значение т дает свое конкретное решение. Например, выбор t = 1 дает ( x, y, z ) = (−4, 11, 1), а t = 3 дает ( x, y, z ) = (4, — 9, −3) и т. Д. Геометрически эта система представляет три плоскости в R 3 , которые пересекаются по линии, и (*) является параметрическим уравнением для этой линии.

Пример 7 дает иллюстрацию системы с бесконечным множеством решений, как возникает этот случай и как записывается решение.Каждая линейная система, имеющая бесконечно много решений, должна содержать хотя бы один произвольный параметр (свободная переменная). После того как расширенная матрица была приведена к эшелонированной форме, количество свободных переменных равно общему количеству неизвестных минус количество ненулевых строк:

Это согласуется с теоремой B выше, которая утверждает, что линейная система с меньшим количеством уравнений, чем неизвестных, если она согласована, имеет бесконечно много решений. Условие «меньше уравнений, чем неизвестных» означает, что количество строк в матрице коэффициентов меньше количества неизвестных.Следовательно, приведенное выше уравнение в рамке подразумевает, что должна быть хотя бы одна свободная переменная. Поскольку такая переменная по определению может принимать бесконечно много значений, система будет иметь бесконечно много решений.

Пример 8 : Найти все решения для системы

Во-первых, обратите внимание, что есть четыре неизвестных, но только три уравнения. Следовательно, если система непротиворечива, гарантировано, что она будет иметь бесконечно много решений, а это состояние характеризуется по крайней мере одним параметром в общем решении.После того, как соответствующая расширенная матрица построена, исключение Гаусса дает

Тот факт, что в эшелонированной форме расширенной матрицы остаются только две ненулевые строки, означает, что 4-2 = 2 переменных свободны:

Следовательно, выбрав y и z в качестве свободных переменных, пусть y = t 1 и z = t 2 . Во второй строке сокращенной расширенной матрицы следует

, а первая строка дает

Таким образом, решения системы имеют вид

, где т 1 т 2 могут принимать любые реальные значения.

Пример 9 : Пусть b = ( b 1 , b 2 , b 3 ) T и пусть A будет матрицей

Для каких значений b 1 , b 2 и b 3 будет ли система A x = b согласованной?

Расширенная матрица для системы A x = b читает

, который гауссовский элиминатин сокращает следующим образом:

Нижняя строка теперь подразумевает, что b 1 + 3 b 2 + b 3 должно быть равно нулю, чтобы эта система была согласованной.Следовательно, в данной системе есть решения (фактически бесконечно много) только для тех векторов-столбцов b = ( b 1 , b 2 , b 3 ) T , для которых b 1 + 3 b 2 + b 3 = 0.

Пример 10 : Решите следующую систему (сравните с Примером 12):

Такая система, как эта, где постоянный член в правой части каждого уравнения равен 0, называется однородной системой .В матричной форме он читает A x = 0 . Поскольку каждая однородная система согласована — поскольку x = 0 всегда является решением, — однородная система имеет либо ровно одно решение ( тривиальное решение , x = 0 ), либо бесконечно много. Сокращение строки матрицы коэффициентов для этой системы уже было выполнено в примере 12. Нет необходимости явно дополнять матрицу коэффициентов столбцом b = 0 , поскольку никакая элементарная операция со строкой не может повлиять на эти нули.То есть, если A ‘является эшелонированной формой A , то операции элементарной строки преобразуют [ A | 0 ] в [ A ′ | 0 ]. По результатам Примера 12,

Поскольку последняя строка снова подразумевает, что z можно принять как свободную переменную, пусть z = t , где t — любое действительное число. Обратная подстановка z = t во вторую строку (- y + 5 z = 0) дает

и обратная подстановка z = t и y = 5 t в первую строку ( x + y -3 z = 0) определяет x :

Следовательно, каждое решение этой системы имеет вид ( x, y, z ) = (−2 t , 5 t, t ), где t — любое действительное число.Растворителей бесконечно много, поскольку каждое действительное значение т дает уникальное частное решение.

Обратите внимание на разницу между набором решений для системы в Примере 12 и здесь. Хотя у обеих была одна и та же матрица коэффициентов A , система в примере 12 была неоднородной ( A x = b , где b 0 ), а здесь — соответствующая однородная система, A x = 0 .Помещая свои решения рядом,

общее решение для Ax = 0 : ( x, y, z ) = (−2 t , 5 t , t )

общее решение для Ax = b : ( x, y, z ) = (−2 t , 5 t , t ) + (−2, 6, 0)

иллюстрирует важный факт:

Теорема C . Общие решения для согласованной неоднородной лиенарной системы, A x = b , равны общему решению соответствующей однородной системы, A x = 0 , плюс частное решение неоднородная система.То есть, если x = x h представляет собой общее решение A x = 0 , то x = x h + x представляет общее решение A x + b , где x — любое конкретное решение (согласованной) неоднородной системы A x = b .

[Техническое примечание: теорема C, которая касается линейной системы , имеет аналог в теории линейных дифференциальных уравнений .Пусть L — линейный дифференциальный оператор; то общее решение разрешимого неоднородного линейного дифференциального уравнения, L (y) = d (где d 0), равно общему решению соответствующего однородного уравнения, L (y) = 0 плюс частное решение неоднородного уравнения. То есть, если y = y h повторно отображает общее решение L (y) = 0, то y = y h + y представляет собой общее решение L (y ) = d , где y — любое частное решение (решаемого) неоднородного линейного уравнения L (y) = d .]

Пример 11 : Определить все решения системы

Запишите расширенную матрицу и выполните следующую последовательность операций:

Поскольку в этой конечной (эшелонированной) матрице остаются только 2 ненулевые строки, есть только 2 ограничения, и, следовательно, 4-2 = 2 из неизвестных — например, y и z — являются свободными переменными. Пусть y = t 1 и z = t 2 .Обратная подстановка y = t 1 и z = t 2 во вторую строку ( x — 3 y + 4 z = 1) дает

Наконец, обратная замена x = 1 + 3 t 1 — 4 2 , y = t 1 и z = t 2 в первую строка (2 w -2 x + y = −1) определяет w :

Следовательно, каждое решение этой системы имеет вид

, где t 1 и t 2 — любые вещественные числа.Другой способ написать решение:

, где т 1 , т 2 R .

Пример 12 : Определите общее решение

, которая является однородной системой, соответствующей неоднородной в примере 11 выше.

Поскольку решение неоднородной системы в примере 11 равно

Теорема C означает, что решение соответствующей однородной системы (где t 1 , t 2 R ) получается из (*), просто отбрасывая конкретное решение, x = (1 / 2,1,0,0) неоднородной системы.

Пример 13 : Докажите теорему A: независимо от ее размера или количества неизвестных, содержащихся в ее уравнениях, линейная система не будет иметь решений, ровно одно решение или бесконечно много решений.

Доказательство . Пусть данная линейная система записана в матричной форме A x = b . Теорема действительно сводится к следующему: если A x = b имеет более одного решения, то на самом деле их бесконечно много.Чтобы установить это, пусть x 1 и x 2 будут двумя разными решениями A x = b . Теперь будет показано, что для любого реального значения t вектор x 1 + t ( x 1 x 2 ) также является решением A x = b ; Поскольку t может принимать бесконечно много различных значений, из этого следует желаемый вывод.Начиная с A x 1 = b и A x 2 ,

Следовательно, x 1 + t ( x 1 x 2 ) действительно является решением A x = b , и теорема доказана.

Систем линейных уравнений: исключение Гаусса

Системы линейных уравнений:
Решение методом исключения Гаусса
(стр. 6 из 7)

Разделы: Определения, Решение по графику, Подстановка, Исключение / добавление, исключение по Гауссу.


Решение трех переменных, линейных систем с тремя уравнениями сложнее, по крайней мере, на начальном этапе, чем решение систем с двумя переменными, потому что требуемые вычисления более грязный. Вы должны быть очень аккуратными в своей работе, и вы должны планируйте использовать много бумаги для заметок. Метод решения этих систем является расширением метода сложения двух переменных, поэтому сделайте конечно ты знаешь это метод хорошо и можно использовать его последовательно правильно.

Хотя метод решения основан на добавлении / исключении, попытка выполнить фактическое добавление имеет тенденцию становится очень запутанным, поэтому существует систематизированный метод решения трех или более переменных системы. Этот метод называется «исключением по Гауссу» (с уравнения заканчиваются так называемой «строковой формой»).

Начнем с простого, и работаем над более сложными примерами.

  • Решите следующие система уравнений.
  • Достаточно легко увидеть как действовать в этом случае. Я просто подставлю обратно значение z -value из третьего уравнения во второе, решите результат для y , а затем штекер z и y в первое уравнение и решите результат для x .

      10 y 3 (3) = 11
      10 y 9 = 11
      10 y = 20
      y = 2

      5x + 4 (2) (3) = 0
      5 x + 8 3 = 0
      5 x + 5 = 0
      5 x = 5
      x = 1

    Тогда решение ( x , y , z ) = (1, 2, 3).

Причина, по которой эта система была Легко решить, что система была «треугольной»; это относится к уравнениям, имеющим форму треугольника, из-за нижних уравнений содержащий только более поздние переменные.

Дело в том, что в этом формат, система проста в решении. А гауссовское исключение — это метод, который мы будем использовать для преобразования систем в эту верхнетреугольную форму, используя операции со строками, которые мы изучили, когда применили метод сложения.

  • Решите следующие система уравнений с использованием исключения Гаусса.
  • Уравнение не решается для переменной, поэтому мне нужно будет выполнить умножение и сложение чтобы упростить эту систему. Чтобы отслеживать свою работу, напишу вниз на каждом шагу, когда я иду. Но я буду делать свои вычисления на бумаге для заметок. Вот как я это сделал:

    Первое, что нужно сделать избавиться от ведущих терминов x в два ряда.А пока я просто посмотрю, какие строки будут легко расчистить; Я могу поменять строки позже, чтобы перевести систему в «верхний треугольной «формы. Нет правила, которое гласит, что я должен использовать x — срок из первой строки, и в этом случае, думаю, будет проще используйте термин x из третьей строки, так как его коэффициент просто «1». Так что я умножу третью строку на 3, и добавьте его в первую строку.Я делаю вычисления на бумаге для заметок:

    … а потом записываю результатов:

    (Когда мы решали системы с двумя переменными, мы могли умножить строку, переписав систему в сторону, а затем добавить. Для этого нет места в система с тремя переменными, поэтому нам и нужна бумага для заметок.) ​​

    Предупреждение: поскольку я не на самом деле ничего не делаю с третьей строкой, я скопировал ее без изменений, в новую матрицу уравнений.Я б / у третий ряд, но я на самом деле не менял Это. Не путайте «использование» с «изменением».

    Чтобы получить меньшие числа для коэффициентов умножу первую строку на половину:

    Теперь умножу третий ряд на 5 и добавьте это ко второму строка. Работаю на бумаге для заметок:

    … а потом записываю результаты: Авторские права Элизабет Стапель 2003-2011 Все права защищены

    Я ничего не делал с первым рядом, поэтому я скопировал его без изменений. Я работал с третий ряд, но я работал только на вторая строка, поэтому вторая строка обновляется, а третья строка копируется более без изменений.

    Хорошо, теперь x — столбец удаляется, за исключением ведущего члена в третьей строке.Так что дальше Приходится работать с колонкой y .

    Предупреждение: С третьего уравнение имеет член x , Я больше не могу использовать его ни в одном из двух других уравнений (или я отменить мой прогресс). Я могу работать с на уравнении, но не на с Это.

    Если я добавлю в два раза больше первого строки во вторую строку, это даст мне ведущую 1 во втором ряду.Я не буду избавились от ведущего y -term во втором ряду, но я его преобразовал (не вмешиваясь дробями) в более простую форму. (Вы должны сохранить обратите внимание на такого рода упрощения.) Сначала я делаю царапину работа:

    … а потом записываю результатов:

Теперь могу использовать второй ряд, чтобы убрать y -term в первом ряду.Вторую строку умножу на 7 и добавить. Сначала я царапаю работа:

… а потом записываю результатов:

Я могу сказать что z сейчас, но для большей точности я разделю первую строку на 43. Затем я переставляю ряды, чтобы придать им верхнетреугольную форму:

Теперь я могу начать процесс обратного решения:

Тогда решение ( x , y , z ) = ( 2, 3, 1 ) .

Примечание: нет ничего священного о шагах, которые я использовал для решения указанной выше системы; там ничего не было особенно о том, как я решил эту систему. Вы могли бы работать в другом упорядочивайте или упрощайте разные строки, и все равно получите правильный ответ. Эти системы достаточно сложны, поэтому вряд ли один правильный способ вычисления ответа. Так что не беспокойтесь о том, «как она знала, что делать дальше? », потому что здесь нет правила.я просто делал все, что пришло мне в голову; Я делал то, что казалось самым простым или что-то еще пришла в голову первая. Не волнуйтесь, если бы вы использовали совершенно другой шаги. Если каждый шаг на этом пути верен, вы придумаете Такой же ответ.


В приведенном выше примере я мог пошли дальше в своих вычислениях и более тщательно проработали строковые операции, удаляя все термины и кроме этого во втором ряду и во всех терминах z кроме того, что в первой строке.Это то, что процесс тогда выглядело так:

Так я могу просто читать от значений x , y , и z , и мне не нужно заморачиваться с обратной заменой. Это более полное метод решения называется «методом исключения Гаусса-Жордана» (с уравнения, попадающие в так называемый «пониженный ряд-эшелон» форма»).Многие тексты доходят до исключения Гаусса, но я всегда было легче продолжать и делать Гаусс-Джордан.

Обратите внимание, что я выполнил две строковые операции сразу на этом последнем шаге перед переключением строк. Пока я не работая с и работая с в том же ряду на том же шаге, это нормально. В этом случае я работал с первой строкой и рабочая по второй и третий ряды.

<< Предыдущая Вверх | 1 | 2 | 3 | 4 | 5 | 6 | 7 | Вернуться к указателю Далее >>

Цитируйте эту статью как:

Стапель, Елизавета. «Системы линейных уравнений, решаемые методом исключения Гаусса». Purplemath
Доступно с https: // www.purplemath.com/modules/systlin6.htm .
Дата обращения [Дата] [Месяц] 2016 г.

M.7 Исключение Гаусса-Джордана | STAT ONLINE

Исключение Гаусса-Жордана — это алгоритм, который можно использовать для решения систем линейных уравнений и нахождения обратной матрицы любой обратимой матрицы. Он полагается на три операции с элементарной строкой , которые можно использовать с матрицей:

  1. Поменять местами две строки
  2. Умножьте одну из строк на ненулевой скаляр.
  3. Добавить или вычесть скалярное кратное одной строки из другой строки.

В качестве примера операции с первой элементарной строкой поменяйте местами 1-ю и 3-ю строки.

\ [\ begin {pmatrix} 4 & 0 & -1 \\ 2 & -2 & 3 \\ 7 & 5 & 0 \ end {pmatrix} \ Rightarrow \ begin {pmatrix} 7 & 5 & 0 \\ 2 & -2 & 3 \\ 4 & 0 & -1 \ end {pmatrix} \]

Для примера операции со второй элементарной строкой умножьте вторую строку на 3.

\ [\ begin {pmatrix} 4 & 0 & -1 \\ 2 & -2 & 3 \\ 7 & 5 & 0 \ end {pmatrix} \ Rightarrow \ begin {pmatrix} 4 & 0 & -1 \\ 6 & -6 & 9 \\ 7 & 5 & 0 \ end {pmatrix} \]

В качестве примера операции с третьей элементарной строкой добавьте дважды первую строку ко второй строке.

\ [\ begin {pmatrix} 4 & 0 & -1 \\ 2 & -2 & 3 \\ 7 & 5 & 0 \ end {pmatrix} \ Rightarrow \ begin {pmatrix} 4 & 0 & -1 \\ 10 & -2 & 1 \\ 7 & 5 & 0 \ end {pmatrix} \]


Редукторный эшелон формы

Целью метода исключения Гаусса-Жордана является использование трех элементарных операций со строками для преобразования матрицы в эшелонированную форму сокращенных строк.Матрица находится в форме сокращенных строк, также известной как каноническая форма строк, , если выполняются следующие условия:

  1. Все строки с нулевыми записями находятся внизу матрицы
  2. Первая ненулевая запись в строке, называемая ведущей записью или опорной точкой , каждой ненулевой строки находится справа от ведущей записи строки над ней.
  3. Начальная запись, также известная как точка поворота, в любой ненулевой строке равна 1.
  4. Все остальные записи в столбце, содержащие в начале 1, являются нулями.

Например,

\ [A = \ begin {pmatrix} 1 & 0 & 0 \\ 0 & 1 & 3 \\ 0 & 0 & 0 \ end {pmatrix}, B = \ begin {pmatrix} 1 & 0 & 0 \\ 0 & 1 & 0 \\ 0 & 0 & 1 \ end {pmatrix}, C = \ begin {pmatrix} 0 & 7 & 3 \\ 1 & 0 & 0 \\ 0 & 0 & 0 \ end {pmatrix}, D = \ begin {pmatrix} 1 & 7 & 3 \\ 0 & 1 & 0 \\ 0 & 0 & 1 \ end {pmatrix} \]

Матрицы A и B представлены в виде эшелона с уменьшенной строкой, а матрицы C и D — нет. C не находится в форме пониженного ряда, поскольку нарушает условия два и три. D не находится в форме пониженного ряда, поскольку нарушает четвертое условие. Кроме того, операции с элементарными строками могут использоваться для уменьшения матрицы D в матрицу B .


Шаги для исключения Гаусса-Джордана

Для выполнения исключения Гаусса-Джордана:

  1. Поменяйте местами строки так, чтобы все строки со всеми нулевыми записями находились внизу
  2. Поменяйте местами строки так, чтобы строка с самой большой левой ненулевой записью была наверху.
  3. Умножьте верхнюю строку на скаляр так, чтобы ведущая запись верхней строки стала 1.
  4. Сложить / вычесть кратные числа верхней строки из других строк, чтобы все остальные записи в столбце, содержащем ведущую запись верхней строки, были равны нулю.
  5. Повторите шаги 2–4 для следующей самой левой ненулевой записи, пока все ведущие записи не станут 1.
  6. Поменяйте местами строки так, чтобы ведущая запись каждой ненулевой строки находилась справа от ведущей записи строки над ней.

Выбранные примеры видео показаны ниже:

Чтобы получить инверсию матрицы n × n A :

  1. Создайте разделенную матрицу \ ((A | I) \), где I — единичная матрица.{-1} = I \).

Матрицы и системы уравнений

6.1 — Матрицы и системы уравнений

Определение матрицы

  • Прямоугольный массив действительных чисел
  • м строк по n столбцов
  • Названо заглавными буквами
  • Первый нижний индекс — строка, второй нижний индекс — столбец

Терминология

  • Матрица с m строками и n столбцами называется матрицей порядка m x n .
  • Квадратная матрица — это матрица с равным количеством строк и столбцов. Поскольку количество строки и столбцы одинаковы, говорят, что он имеет порядок n .
  • Главной диагональю квадратной матрицы являются элементы от верхнего левого угла до нижнего правого угла. матрица.
  • Матрица-строка — это матрица, содержащая только одну строку.
  • Матрица столбцов — это матрица, имеющая только один столбец.
  • Матрица только с одной строкой или одним столбцом называется вектором.

Преобразование систем линейных уравнений в Матрицы

Каждое уравнение в системе превращается в строку. Каждая переменная в система становится колонной. Переменные отбрасываются, а коэффициенты помещаются в матрицу. Если правая часть включена, это называется расширенной матрицей. Если правая сторона не указана, это называется матрицей коэффициентов.

Система линейных уравнений …

 х + у - г = 1
3х - 2у + г = 3
4x + y - 2z = 9 

становится расширенной матрицей…

х y z справа
1 1–1 1
3-2 1 3
4 1-2 9

Элементарные операции со строками

Элементарные операции со строками — это операции, которые могут быть выполнены с матрицей, которая даст эквивалентная строка матрица.Если матрица является расширенной матрицей, построенной из системы линейных уравнений, то эквивалентная строка матрица будет иметь то же решение, что и исходная матрица.

При работе с системами линейных уравнений вы могли выполнять три операции. что не повлияет на набор решений.

  1. Поменяйте местами два уравнения.
  2. Умножьте уравнение на ненулевую константу.
  3. Умножьте уравнение на ненулевую константу и добавьте его к другому уравнению, заменив это уравнение.

Когда система линейных уравнений преобразуется в расширенную матрицу, каждое уравнение становится строка. Итак, теперь есть три элементарные операции со строками, которые производят эквивалент строки матрица.

  1. Перестановка двухрядная
  2. Умножить строку на ненулевую константу
  3. Умножьте строку на ненулевую константу и добавьте ее в другую строку, заменив эту строку.

Формы рядов-эшелонов и сокращенных рядов-эшелонов

Это эквивалентные строкам формы матрицы.Несложно решить систему линейных уравнений когда матрицы находятся в одной из этих форм.

Форма рядного эшелона

Матрица находится в виде эшелона строк, когда выполняются следующие условия.

  1. Если есть строка со всеми нулями, то она находится внизу матрицы.
  2. Первый ненулевой элемент любой строки — это единица. Этот элемент называется ведущим.
  3. Первая строка любой строки находится справа от первой строки предыдущей строки.
Примечания
  • Ведущий в строке не обязательно должен быть рядом с справа от ведущей строки предыдущий ряд.
  • Матрица в виде эшелона строк будет иметь нули под ведущими.
  • Метод исключения Гаусса переводит матрицу в форму строки-эшелон, а затем выполняется обратная подстановка. требуется, чтобы завершить поиск решений системы.
  • Форма строки-эшелон матрицы не обязательно уникальна.

Уменьшенная форма рядка-эшелон

Матрица находится в сокращенной форме строки-эшелона, когда выполняются все условия формы строка-эшелон. и все элементы выше и ниже, ведущие равны нулю.

  1. Если есть строка со всеми нулями, то она находится внизу матрицы.
  2. Первый ненулевой элемент любой строки — это единица. Этот элемент называется ведущим.
  3. Первая строка любой строки находится справа от первой строки предыдущей строки.
  4. Все элементы выше и ниже ведущего равны нулю.
Примечания
  • Ведущий в строке не обязательно должен быть рядом с справа от ведущей строки предыдущий ряд.
  • Матрица в виде эшелона строк будет иметь нули как над, так и под ведущими.
  • Метод исключения Гаусса-Джордана переводит матрицу в упрощенную форму строки-эшелона.
  • Для завершения поиска решений системы обратная подстановка не требуется.
  • Редуцированная строка-эшелонированная форма матрицы уникальна.

Исключение по Гауссу

  • Запишите систему линейных уравнений в виде расширенной матрицы
  • Выполните элементарные операции со строками, чтобы преобразовать матрицу в эшелонированную форму строки
  • Преобразуйте матрицу обратно в систему линейных уравнений
  • Используйте обратную замену, чтобы получить все ответы

Гаусс-Джордан Ликвидация

  • Запишите систему линейных уравнений в виде расширенной матрицы
  • Выполните элементарные операции со строками, чтобы преобразовать матрицу в сокращенную форму строки-эшелона
  • Преобразуйте матрицу обратно в систему линейных уравнений
  • Обратной замены не требуется

Поворотный

  • Поворот — это процесс, который автоматизирует операции со строками, необходимые для размещения матрицы в рядный эшелон или редуцированный рядный эшелон
  • В частности, при повороте элементы выше или ниже ведущей единицы превращаются в нули

Типы решений

Существует три типа решений, которые возможны при решении системы линейных уравнений.

Независимый
  • Согласованный
  • Уникальное решение
  • Матрица с сокращенной строкой имеет такое же количество ненулевых строк, что и переменные
  • Левая часть обычно представляет собой единичную матрицу, но не обязательно
  • Для получения независимого решения должно быть как минимум столько же уравнений, сколько переменных.
х y z справа
1 0 0 3
0 1 0 1
0 0 1 2

Когда вы конвертируете расширенную матрицу обратно в форму уравнения, вы получаете x = 3, y = 1 и z = 2.

Зависимые
  • Согласованный
  • Множество решений
  • Запишите ответ в параметрической форме
  • Матрица с сокращенной строкой содержит больше переменных, чем ненулевых строк
  • Ряд нулей быть не обязательно, но обычно он есть.
  • Это также может произойти, когда уравнений меньше, чем переменных.
х y z справа
1 0 3 4
0 1-2 3
0 0 0 0

Первое уравнение будет x + 3z = 4.Решение относительно x дает x = 4 — 3z.

Второе уравнение будет y — 2z = 3. Решение для y дает y = 3 + 2z.

Столбец z не очищается (все нули, кроме одно число), поэтому другие переменные будут определены через z. Следовательно, z будет параметром t и решение …

x = 4 — 3t, y = 3 + 2t, z = t

Несоответствие
  • Нет решения
  • Матрица с сокращенной строкой имеет строку нулей слева, но правая часть не равна нулю.
х y z справа
1 0 3 4
0 1-2 3
0 0 0 2

Тут решения нет.Вы можете записать это как нулевой набор Ø, пустой набор {} или нет решения.

Решающих систем с исключением Гаусса — College Algebra

Цели обучения

В этом разделе вы:

  • Напишите расширенную матрицу системы уравнений.
  • Напишите систему уравнений из расширенной матрицы.
  • Выполняет операции со строками в матрице.
  • Решите систему линейных уравнений с помощью матриц.

Немецкий математик Карл Фридрих Гаусс (1777–1855).

Карл Фридрих Гаусс жил в конце 18 — начале 19 века, но до сих пор считается одним из самых плодовитых математиков в истории. Его вклад в математику и физику охватывает такие области, как алгебра, теория чисел, анализ, дифференциальная геометрия, астрономия и оптика. Его открытия в области теории матриц изменили способ работы математиков за последние два столетия.

Мы впервые столкнулись с методом исключения Гаусса в системах линейных уравнений: две переменные.В этом разделе мы еще раз вернемся к этой технике решения систем, на этот раз с использованием матриц.

Запись расширенной матрицы системы уравнений

Матрица может служить средством представления и решения системы уравнений. Чтобы выразить систему в матричной форме, мы извлекаем коэффициенты переменных и констант, и они становятся элементами матрицы. Мы используем вертикальную линию, чтобы отделить записи коэффициентов от констант, по сути заменяя знаки равенства.Когда система написана в такой форме, мы называем ее расширенной матрицей.

Например, рассмотрим следующую систему уравнений.

Мы можем записать эту систему в виде расширенной матрицы:

Мы также можем написать матрицу, содержащую только коэффициенты. Это называется матрицей коэффициентов.

Система уравнений три на три, например

имеет матрицу коэффициентов

и представлена ​​расширенной матрицей

Обратите внимание, что матрица написана таким образом, что переменные выстраиваются в свои собственные столбцы: x -термов идет в первый столбец, -терминов во втором столбце и z -термов в третьем столбце.Очень важно, чтобы каждое уравнение было написано в стандартной форме, чтобы переменные совпадали. Если в уравнении отсутствует член переменной, коэффициент равен 0.

Для данной системы уравнений напишите расширенную матрицу.

  1. Запишите коэффициенты членов x в виде чисел в первом столбце.
  2. Запишите коэффициенты членов y как числа во втором столбце.
  3. Если имеется z -термов, запишите коэффициенты в виде чисел в третьем столбце.
  4. Нарисуйте вертикальную линию и напишите константы справа от нее.

Написание расширенной матрицы для системы уравнений

Напишите расширенную матрицу для данной системы уравнений.

Расширенная матрица отображает коэффициенты переменных и дополнительный столбец для констант.

Запишите расширенную матрицу данной системы уравнений.

Написание системы уравнений из расширенной матрицы

Мы можем использовать расширенные матрицы, чтобы помочь нам решать системы уравнений, потому что они упрощают операции, когда системы не обременены переменными.Однако важно понимать, как переключаться между форматами, чтобы поиск решений был более плавным и интуитивно понятным. Здесь мы будем использовать информацию в расширенной матрице, чтобы записать систему уравнений в стандартной форме.

Напишите систему уравнений из расширенной матрицы.

Выполнение операций со строками в матрице

Теперь, когда мы можем писать системы уравнений в форме расширенной матрицы, мы рассмотрим различные операции со строками, которые могут выполняться с матрицей, такие как сложение, умножение на константу и перестановка строк.

Выполнение строковых операций над матрицей — это метод, который мы используем для решения системы уравнений. Чтобы решить систему уравнений, мы хотим преобразовать матрицу в форму строки-эшелона, в которой есть единицы вниз по главной диагонали от верхнего левого угла до нижнего правого угла и нули в каждой позиции ниже главной диагонали. как показано.

Мы используем операции со строками, соответствующие операциям с уравнениями, чтобы получить новую матрицу, эквивалентную строкам в более простой форме.Вот рекомендации по получению формы рядного эшелона.

  1. В любой ненулевой строке первым ненулевым числом является 1. Оно называется ведущим 1.
  2. Любые нулевые строки помещаются внизу матрицы.
  3. Любая ведущая 1 находится ниже и правее предыдущей ведущей 1.
  4. Любой столбец, в котором в начале стоит 1, имеет нули во всех остальных позициях в столбце.

Чтобы решить систему уравнений, мы можем выполнить следующие операции со строками, чтобы преобразовать матрицу коэффициентов в форму ряда строк и выполнить обратную подстановку, чтобы найти решение.

  1. Поменять местами ряды. (Обозначение 🙂
  2. Умножьте строку на константу. (Обозначение 🙂
  3. Добавить произведение одной строки на константу к другой строке. (Замечание:

Каждая из строковых операций соответствует операциям, которые мы уже научились решать системы уравнений с тремя переменными. С помощью этих операций есть несколько ключевых ходов, которые быстро достигнут цели написания матрицы в виде эшелона строк. Чтобы получить матрицу в виде эшелона строк для поиска решений, мы используем метод исключения Гаусса, который использует операции со строками для получения 1 в качестве первой записи, чтобы строку 1 можно было использовать для преобразования оставшихся строк.

Исключение по Гауссу

Метод исключения Гаусса относится к стратегии, используемой для получения многоуровневой формы матрицы. Цель состоит в том, чтобы записать матрицу с номером 1 в качестве записи по главной диагонали и иметь все нули внизу.

Первый шаг стратегии Гаусса включает получение 1 в качестве первой записи, так что строка 1 может использоваться для изменения строк ниже.

Учитывая расширенную матрицу, выполните операции со строками для получения формы «строка-эшелон».

  1. Первое уравнение должно иметь старший коэффициент 1. При необходимости поменяйте местами строки или умножьте на константу.
  2. Используйте операции со строками, чтобы получить нули в первом столбце под первой записью 1.
  3. Используйте операции со строками, чтобы получить 1 в строке 2, столбце 2.
  4. Используйте операции со строками, чтобы получить нули в столбце 2 под записью 1.
  5. Используйте операции со строками, чтобы получить 1 в строке 3, столбец 3.
  6. Продолжайте этот процесс для всех строк, пока в каждой записи по главной диагонали не будет 1, а внизу будут только нули.
  7. Если какие-либо строки содержат все нули, поместите их внизу.
Решение системы методом исключения Гаусса

Решите данную систему методом исключения Гаусса.

Решите данную систему методом исключения Гаусса.

Использование исключения Гаусса для решения системы уравнений

Используйте метод исключения Гаусса для решения данной системы уравнений.

Решение зависимой системы

Решите систему уравнений.

Выполнение операций со строками в расширенной матрице 3 × 3 для получения формы Row-Echelon

Выполняет операции со строками с заданной матрицей, чтобы получить форму строки-эшелона.

Запишите систему уравнений в виде строк.

Решение системы линейных уравнений с использованием матриц

Мы видели, как написать систему уравнений с расширенной матрицей, а затем как использовать строковые операции и обратную подстановку для получения строчно-эшелонированной формы.Теперь мы перейдем на шаг дальше от строковой формы, чтобы решить систему линейных уравнений 3 на 3. Общая идея состоит в том, чтобы исключить все переменные, кроме одной, с помощью операций со строками, а затем выполнить обратную замену для поиска других переменных.

Решение системы линейных уравнений с использованием матриц

Решите систему линейных уравнений с помощью матриц.

Решение зависимой системы линейных уравнений с использованием матриц

Решите следующую систему линейных уравнений, используя матрицы.

Решите систему, используя матрицы.

Можно ли решить любую систему линейных уравнений методом исключения Гаусса?

Да, система линейных уравнений любого размера может быть решена методом исключения Гаусса.

Для данной системы уравнений решите с помощью матриц с помощью калькулятора.

  1. Сохранить расширенную матрицу как матричную переменную
  2. Используйте функцию ref ( в калькуляторе, вызывая каждую матричную переменную по мере необходимости.

Решение систем уравнений с матрицами с помощью калькулятора

Решите систему уравнений.

Применение матриц 2 × 2 к финансам

Кэролайн инвестирует в общей сложности 12 000 фунтов стерлингов в две муниципальные облигации, одна из которых выплачивает 10,5% годовых, а другая — 12%. Годовой процент, полученный по двум инвестициям в прошлом году, составил 1335 фунтов стерлингов. Сколько было вложено по каждой ставке?

Применение матриц 3 × 3 к финансам

Ava инвестирует в общей сложности 10 000 фунтов стерлингов в три счета, один из которых платит 5% годовых, другой — 8%, а третий — 9%.Годовой процент, полученный по трем инвестициям в прошлом году, составил 770 фунтов стерлингов. Сумма, вложенная под 9%, была вдвое больше, чем сумма, вложенная под 5%. Сколько было вложено по каждой ставке?

У нас есть система трех уравнений с тремя переменными. Пусть будет сумма, вложенная под 5%, пусть будет сумма, вложенная под 8%, пусть будет сумма, вложенная под 9%. Таким образом,

В качестве матрицы имеем

Теперь мы выполняем исключение Гаусса, чтобы получить форму строки-эшелон.

Третья строка сообщает usthus

Вторая строка говорит нам Подставляя, мы получаем

Первая строка говорит нам о замене и получаем

Ответ: 3000 евро вложены под 5%, 1000 евро вложены под 8% и 6000 евро инвестированы под 9%.

Небольшая обувная компания взяла ссуду в размере 1 500 000 фунтов стерлингов для расширения своих запасов. Часть денег была взята под 7%, часть — под 8%, часть — под 10%. Сумма займа под 10% в четыре раза превышала сумму займа под 7%, а годовая процентная ставка по всем трем займам составляла 130 500 фунтов стерлингов. Используйте матрицы, чтобы найти сумму займа по каждой ставке.

? 150 000 при 7%, 750 000 фунтов стерлингов при 8%, 600 000 фунтов стерлингов при 10%

Ключевые понятия

  • Расширенная матрица — это матрица, которая содержит коэффициенты и константы системы уравнений.См. (Рисунок).
  • Матрица, дополненная постоянным столбцом, может быть представлена ​​как исходная система уравнений. См. (Рисунок).
  • Операции со строками включают в себя умножение строки на константу, добавление одной строки к другой строке и перестановку строк.
  • Мы можем использовать метод исключения Гаусса для решения системы уравнений. См. (Рисунок), (Рисунок) и (Рисунок).
  • Операции со строками выполняются над матрицами для получения формы «строка-эшелон». См. (Рисунок).
  • Чтобы решить систему уравнений, запишите ее в форме расширенной матрицы.Выполните операции со строками, чтобы получить форму эшелона строк. Обратно-заменитель, чтобы найти решения. См. (Рисунок) и (Рисунок).
  • Калькулятор можно использовать для решения систем уравнений с использованием матриц. См. (Рисунок).
  • Многие реальные проблемы можно решить с помощью расширенных матриц. См. (Рисунок) и (Рисунок).

Упражнения по разделам

Вербальный

Можно ли записать любую систему линейных уравнений в виде расширенной матрицы? Объясните, почему да или почему нет. Объясните, как написать эту расширенную матрицу.

Да. Для каждой строки коэффициенты переменных записываются поперек соответствующей строки и помещается вертикальная черта; затем константы помещаются справа от вертикальной полосы.

Можно ли записать любую матрицу в виде системы линейных уравнений? Объясните, почему да или почему нет. Объясните, как написать эту систему уравнений.

Есть только один правильный метод использования операций со строками в матрице? Попытайтесь объяснить две различные операции со строками, которые можно выполнить для расширенной матрицы

.

Нет, существует множество правильных методов использования строковых операций над матрицей.Есть два возможных способа: (1) Поменять местами строки 1 и 2. Затем (2) Разделить строку 1 на 9.

Можно ли решить матрицу с нулевым элементом на диагонали? Объясните, почему да или почему нет. Что бы вы сделали, чтобы исправить ситуацию?

Может ли матрица с 0 элементами для всей строки иметь одно решение? Объясните, почему да или почему нет.

Нет. Матрица с 0 элементами для всей строки будет иметь либо ноль, либо бесконечно много решений.

Алгебраические

Для следующих упражнений напишите расширенную матрицу линейной системы.

Для следующих упражнений запишите линейную систему из расширенной матрицы.

Для следующих упражнений решите систему методом исключения Гаусса.

Расширения

Для следующих упражнений используйте метод исключения Гаусса для решения системы.

Реальные приложения

Для следующих упражнений настройте расширенную матрицу, описывающую ситуацию, и найдите желаемое решение.

Ежедневно в магазине кексов продается 5 000 кексов со вкусом шоколада и ванили. Если вкус шоколада в 3 раза популярнее, чем аромат ванили, сколько кексов продается в день?

В конкурирующем магазине кексов ежедневно продаются кексы на сумму 4520 фунтов стерлингов.Шоколадные кексы стоят 2,25 евро, а кексы из красного бархата — 1,75 евро. Если общее количество кексов, проданных в день, составляет 2200, сколько кексов каждого вкуса продается каждый день?

860 красный бархат, 1340 шоколад

Вы вложили 10 000 евро в два счета: один с простой процентной ставкой 3%, а другой — с процентной ставкой 2,5%. Если ваша общая сумма процентов по истечении одного года составила 283,50 фунтов стерлингов, какая сумма была на каждом счете по истечении года?

Вы вложили 2300 евро на счет 1 и 2700 евро на счет 2.Если общая сумма процентов по истечении одного года составляет 254 евро, а на счете 2 процентная ставка в 1,5 раза выше, чем на счете 1, каковы процентные ставки? Предположим простые процентные ставки.

4% на счет 1, 6% на счет 2

Bikes’R’Us производит велосипеды по 250 фунтов стерлингов. Производитель обошелся в 180 фунтов стерлингов за велосипед плюс стартовый взнос в размере 3500 фунтов стерлингов. Через сколько проданных велосипедов производитель выйдет на уровень безубыточности?

Крупный магазин бытовой техники рассматривает возможность приобретения пылесосов у небольшого производителя.Магазин сможет приобрести пылесосы по 86 фунтов стерлингов каждый, со стоимостью доставки 9 200 фунтов стерлингов, независимо от того, сколько пылесосов продано. Если магазин должен начать получать прибыль после продажи 230 единиц, сколько они должны взимать плату за пылесосы?

Три самых популярных вкуса мороженого — это шоколад, клубника и ваниль, составляющие 83% вкусов, продаваемых в магазине мороженого. Если ваниль продается на 1% больше, чем в два раза больше клубники, а шоколад продается на 11% больше, чем ваниль, сколько в общем потреблении мороженого приходится на ароматы ванили, шоколада и клубники?

В магазине мороженого возрастает спрос на три вкуса.В прошлом году банановое, тыквенное и мороженое с каменистой дорогой составили 12% от общего объема продаж мороженого. В этом году на те же три вида мороженого пришлось 16,9% продаж мороженого. Продажи по каменистой дороге увеличились вдвое, продажи бананов увеличились на 50%, а продажи тыквы — на 20%. Если у мороженого по каменистой дороге было на один процент меньше продаж, чем у бананового мороженого, узнайте процент продаж мороженого, произведенного каждым отдельным мороженым в прошлом году.

Банан — 3%, тыква — 7%, а каменистая дорога — 2%

В пакете ореховой смеси кешью, фисташки и миндаль.Всего в сумке 1000 орехов, а миндаля на 100 меньше, чем фисташек. Кешью весит 3 г, фисташки — 4 г, миндаль — 5 г. Если мешок весит 3,7 кг, узнайте, сколько орехов каждого вида в нем.

В пакете ореховой смеси кешью, фисташки и миндаль. Изначально в сумке было 900 орехов. Было съедено 30% миндаля, 20% кешью и 10% фисташек, и теперь в сумке осталось 770 орехов. Изначально кешью было на 100 штук больше, чем миндаля.Для начала выясните, сколько орехов каждого типа было в пакете.

100 миндальных орехов, 200 кешью, 600 фисташек

Глоссарий

расширенная матрица
матрица коэффициентов, примыкающая к столбцу констант, разделенному вертикальной линией в скобках матрицы
матрица коэффициентов
матрица, содержащая только коэффициенты из системы уравнений
Исключение Гаусса
с использованием элементарных операций со строками для получения матрицы в виде строки-эшелона
главная диагональ
записей из левого верхнего угла по диагонали в правый нижний угол квадратной матрицы
рядная форма
после выполнения операций со строками матричная форма, содержащая единицы по главной диагонали и нули в каждом пробеле ниже диагонали
эквивалент строки
две матрицы и эквивалентны строкам, если одна может быть получена из другой путем выполнения основных операций со строками
строковые операции
: добавление одной строки к другой, умножение строки на константу, перестановка строк и т. Д. С целью получения формы «строка-эшелон»

Обращение матрицы с использованием исключения Гаусса-Джордана

М.Борн

В этом разделе мы увидим, как работает метод исключения Гаусса-Жордана, на примерах.

Вы можете повторно загружать эту страницу сколько угодно раз и каждый раз получать новый набор чисел. Вы также можете выбрать матрицу другого размера (внизу страницы).

(Если вам сначала нужна дополнительная информация, вернитесь к «Введение в матрицы»).

Выберите размер матрицы, который вас интересует, и нажмите кнопку.

Матрица A:

Пример, сгенерированный случайным образом, показан ниже.

Телефонных пользователей

ПРИМЕЧАНИЕ: Если вы пользуетесь телефоном, вы можете прокрутить любую матрицу шириной на этой странице вправо или влево, чтобы увидеть все выражение.

Пример (3 × 3)

Найдите матрицу, обратную матрице A , используя метод исключения Гаусса-Жордана.

A = 11 9 6
7 8 13
14 10 2

Наша процедура

Запишем матрицу A слева и матрицу идентичности I справа, разделенные пунктирной линией, как показано ниже.Результат называется расширенной матрицей .

Мы включили номера строк, чтобы было понятнее.

1 0 0 Ряд [1]
0 1 0 ряд [2]
0 0 1 ряд [3]

Затем мы выполняем несколько операций со строками над двумя матрицами, и наша цель — получить единичную матрицу на левом , например:

??? Ряд [1]
??? ряд [2]
??? ряд [3]

(Технически мы сокращаем матрицу A до сокращенной формы эшелона строк , также называемой канонической формой строк ).

Результирующая матрица справа будет матрицей, обратной для A .

Наша процедура операций со строками выглядит следующим образом:

  1. Получим «1» в верхнем левом углу, разделив первую строку
  2. Тогда мы получим «0» в оставшейся части первого столбца
  3. Затем нам нужно получить «1» во второй строке, втором столбце
  4. Затем мы делаем все остальные записи во втором столбце «0».

Продолжаем так до тех пор, пока слева не останется единичная матрица.

Давайте теперь продолжим и найдем обратное.

Решение

Начнем с:

1 0 0 Ряд [1]
0 1 0 ряд [2]
0 0 1 ряд [3]

Новый ряд [1]

Разделите строку [1] на 11 (чтобы получить «1» в нужной позиции):

Это дает нам:

1 0.8182 0,5455
7 8 13
14 10 2
0,0909 0 0 Ряд [1]
0 1 0 ряд [2]
0 0 1 ряд [3]

Новый ряд [2]

Ряд [2] — 7 × Ряд [1] (чтобы получить 0 в желаемой позиции):

7 — 7 × 1 = 0
8 — 7 × 0.8182 = 2,2727
13 — 7 × 0,5455 = 9,1818
0 — 7 × 0,0909 = -0,6364
1 — 7 × 0 = 1
0 — 7 × 0 = 0

Это дает нам новую строку [2]:

1 0,8182 0,5455
0 2,2727 9,1818
14 10 2
0,0909 0 0 Ряд [1]
-0.6364 1 0 ряд [2]
0 0 1 ряд [3]

Новый ряд [3]

Ряд [3] — 14 × Ряд [1] (чтобы получить 0 в желаемой позиции):

14 — 14 × 1 = 0
10 — 14 × 0,8182 = -1,4545
2 — 14 × 0,5455 = -5,6364
0 — 14 × 0,0909 = -1,2727
0 — 14 × 0 = 0
1 — 14 × 0 = 1

Это дает нам новую строку [3]:

1 0.8182 0,5455
0 2,2727 9,1818
0 -1,4545 -5,6364
0,0909 0 0 Ряд [1]
-0,6364 1 0 ряд [2]
-1,2727 0 1 ряд [3]

Новый ряд [2]

Разделите строку [2] на 2.2727 (чтобы получить «1» в желаемой позиции):

Это дает нам:

1 0,8182 0,5455
0 1 4,04
0 -1,4545 -5,6364
0,0909 0 0 Ряд [1]
-0,28 0.44 0 ряд [2]
-1,2727 0 1 ряд [3]

Новый ряд [1]

Ряд [1] — 0,8182 × Ряд [2] (чтобы дать нам 0 в желаемой позиции):

1 — 0,8182 × 0 = 1
0,8182 — 0,8182 × 1 = 0
0,5455 — 0,8182 × 4,04 = -2,76
0,0909 — 0,8182 × -0,28 = 0,32
0 — 0,8182 × 0,44 = -0,36
0 — 0,8182 × 0 = 0

Это дает нам новую строку [1]:

1 0 -2.76
0 1 4,04
0 -1,4545 -5,6364
0,32 -0,36 0 Ряд [1]
-0,28 0,44 0 ряд [2]
-1,2727 0 1 ряд [3]

Новый ряд [3]

Ряд [3] — -1.4545 × Ряд [2] (чтобы дать нам 0 в желаемой позиции):

0 — -1,4545 × 0 = 0
-1,4545 — -1,4545 × 1 = 0
-5,6364 — -1,4545 × 4,04 = 0,24
-1,2727 — -1,4545 × -0,28 = -1,68
0 — -1,4545 × 0,44 = 0,64
1 — -1,4545 × 0 = 1

Это дает нам новую строку [3]:

1 0 -2,76
0 1 4,04
0 0 0.24
0,32 -0,36 0 Ряд [1]
-0,28 0,44 0 ряд [2]
-1,68 0,64 1 ряд [3]

Новый ряд [3]

Разделите строку [3] на 0,24 (чтобы получить «1» в нужной позиции):

Это дает нам:

1 0 -2.76
0 1 4,04
0 0 1
0,32 -0,36 0 Ряд [1]
-0,28 0,44 0 ряд [2]
-7 2,6667 4,1667 ряд [3]

Новый ряд [1]

Ряд [1] — -2.76 × Ряд [3] (чтобы дать нам 0 в желаемой позиции):

1 — -2,76 × 0 = 1
0 — -2,76 × 0 = 0
-2,76 — -2,76 × 1 = 0
0,32 — -2,76 × -7 = -19
-0,36 — -2,76 × 2,6667 = 7
0 — -2,76 × 4,1667 = 11,5

Это дает нам новую строку [1]:

-19 7 11,5 Ряд [1]
-0,28 0,44 0 ряд [2]
-7 2.6667 4,1667 ряд [3]

Новый ряд [2]

Ряд [2] — 4,04 × Ряд [3] (чтобы получить 0 в желаемой позиции):

0 — 4,04 × 0 = 0
1 — 4,04 × 0 = 1
4,04 — 4,04 × 1 = 0
-0,28 — 4,04 × -7 = 28
0,44 — 4,04 × 2,6667 = -10,333
0 — 4,04 × 4,1667 = -16,833

Это дает нам новую строку [2]:

-19 7 11,5 Ряд [1]
28 -10.333 -16,833 ряд [2]
-7 2,6667 4,1667 ряд [3]

Мы достигли нашей цели по созданию матрицы идентичности слева. Таким образом, мы можем заключить, что инверсия матрицы A является правой частью расширенной матрицы:

A -1 =-19 7 11.5
28 -10,333 -16,833
-7 2,6667 4,1667

Примечания

  1. В приведенном выше объяснении показаны все шаги. Человек обычно может пойти несколькими путями. Кроме того, иногда в правильной позиции уже есть «1» или «0», и в таких случаях нам не нужно ничего делать для этого шага.
  2. Всегда записывайте, что вы делаете на каждом этапе — очень легко заблудиться!
  3. Я показал результаты с точностью до 4 знаков после запятой, но с максимальной точностью использовалась повсюду.Имейте в виду, что небольшие ошибки округления будут накапливаться во всей задаче. Всегда используйте полную точность калькулятора! (Используйте всю память вашего калькулятора.)
  4. Очень иногда возникают странные результаты из-за внутреннего представления чисел компьютером. То есть он может хранить «1» как 0,999999999872.

Смотрите еще?

Вы можете вернуться к началу страницы и выбрать другой пример.

1.3: Исключение Гаусса — Математика LibreTexts

Работа, которую мы проделали в предыдущем разделе, всегда найдет решение системы.В этом разделе мы рассмотрим менее громоздкий способ поиска решений. Сначала мы представим линейную систему с расширенной матрицей . Матрица — это просто прямоугольный массив чисел. Размер или размерность матрицы определяется как \ (m \ times n \), где \ (m \) — количество строк, а \ (n \) — количество столбцов. Чтобы построить расширенную матрицу из линейной системы, мы создаем матрицу коэффициентов из коэффициентов переменных в системе, а также постоянную матрицу из констант.Коэффициенты из одного уравнения системы составляют одну строку расширенной матрицы.

Например, рассмотрим линейную систему в примере [exa: решение системы с элементарными опциями] \ [\ begin {array} {c} x + 3y + 6z = 25 \\ 2x + 7y + 14z = 58 \\ 2y + 5z = 19 \ end {array} \] Эта система может быть записана как расширенная матрица следующим образом \ [\ left [\ begin {array} {rrr | r} 1 & 3 & 6 & 25 \\ 2 & 7 & 14 & 58 \\ 0 & 2 & 5 & 19 \ end {array} \ right] \]

Обратите внимание, что он содержит точно такую ​​же информацию, что и исходная система.Здесь подразумевается, что первый столбец содержит коэффициенты из \ (x \) в каждом уравнении в порядке \ (\ left [\ begin {array} {r} 1 \\ 2 \\ 0 \ end {array} \ right]. \) Аналогичным образом мы создаем столбец из коэффициентов на \ (y \) в каждом уравнении, \ (\ left [\ begin {array} {r} 3 \\ 7 \\ 2 \ end {array} \ right] \) и столбец из коэффициентов на \ (z \) в каждом уравнении, \ (\ left [\ begin {array} {r} 6 \\ 14 \\ 5 \ end {array} \ right]. \ ) Для системы, состоящей из более чем трех переменных, мы продолжим таким же образом построение столбца для каждой переменной.Точно так же для системы из менее чем трех переменных мы просто строим столбец для каждой переменной.

Наконец, мы строим столбец из констант уравнений, \ (\ left [\ begin {array} {r} 25 \\ 58 \\ 19 \ end {array} \ right]. \)

Строки расширенной матрицы соответствуют уравнениям в системе. Например, верхняя строка в расширенной матрице \ (\ left [\ begin {array} {rrrrr} 1 & 3 & 6 & | & 25 \ end {array} \ right] \) соответствует уравнению \ [x + 3у + 6z = 25.\]

Рассмотрим следующее определение.

Определение \ (\ PageIndex {1} \): Расширенная матрица линейной системы

Для линейной системы вида \ [\ begin {array} {c} a_ {11} x_ {1} + \ cdots + a_ {1n} x_ {n} = b_ {1} \\ \ vdots \\ a_ {m1} x_ {1} + \ cdots + a_ {mn} x_ {n} = b_ {m} \ end {array} \], где \ (x_ {i} \) — переменные, а \ (a_ {ij } \) и \ (b_ {i} \) являются константами, расширенная матрица этой системы задается как \ [\ left [\ begin {array} {rrr | r} a_ {11} & \ cdots & a_ {1n } & b_ {1} \\ \ vdots & & \ vdots & \ vdots \\ a_ {m1} & \ cdots & a_ {mn} & b_ {m} \ end {array} \ right] \]

Теперь рассмотрим элементарные операции в контексте расширенной матрицы.Элементарные операции в Definition [def: elementaryoperations] можно использовать для строк так же, как мы использовали их ранее для уравнений. Изменения в системе уравнений в результате элементарной операции эквивалентны изменениям в расширенной матрице в результате соответствующей операции со строкой. Обратите внимание, что из теоремы [thm: elementaryoperationsandsolns] следует, что любые элементарные операции со строками, используемые в расширенной матрице, не изменят решение соответствующей системы уравнений.Теперь мы формально определим элементарные операции со строками. Это ключевой инструмент , который мы будем использовать для поиска решений систем уравнений.

Определение \ (\ PageIndex {2} \): элементарные операции со строками

элементарных операций со строками (также известных как операций со строками ) состоят из следующих

  1. Переключить два ряда.
  2. Умножьте строку на ненулевое число.
  3. Заменить строку любым числом, кратным другой добавленной к ней строке.

Вспомните, как мы решили Пример [например: решение системы с элементарными операциями]. Мы можем сделать те же шаги, что и выше, за исключением того, что теперь в контексте расширенной матрицы и с использованием строковых операций. Расширенная матрица этой системы: \ [\ left [\ begin {array} {rrr | r} 1 & 3 & 6 & 25 \\ 2 & 7 & 14 & 58 \\ 0 & 2 & 5 & 19 \ end { array} \ right] \] Таким образом, первым шагом в решении системы, заданной командой [resolveasystem1], было бы взять \ (\ left (-2 \ right) \), умноженное на первую строку расширенной матрицы, и добавить ее ко второй строка, \ [\ left [\ begin {array} {rrr | r} 1 & 3 & 6 & 25 \\ 0 & 1 & 2 & 8 \\ 0 & 2 & 5 & 19 \ end {array} \ right] \] Обратите внимание, как это соответствует [resolveasystem2].Затем возьмите \ (\ left (-2 \ right) \), умноженный на вторую строку, и добавьте к третьей, \ [\ left [\ begin {array} {rrr | r} 1 & 3 & 6 & 25 \\ 0 & 1 & 2 & 8 \\ 0 & 0 & 1 & 3 \ end {array} \ right] \] Эта расширенная матрица соответствует системе \ [\ begin {array} {c} x + 3y + 6z = 25 \\ y + 2z = 8 \\ z = 3 \ end {array} \], что то же самое, что и [resolveasystem3]. Обратной заменой вы получите решение \ (x = 1, y = 2, \) и \ (z = 3. \)

Посредством систематической процедуры операций со строками мы можем упростить расширенную матрицу и перенести ее в форму эшелон строк или сокращенную форму строки-эшелон, которую мы определим далее.Эти формы используются для поиска решений системы уравнений, соответствующей расширенной матрице.

В следующих определениях термин , ведущая запись относится к первой ненулевой записи строки при сканировании строки слева направо.

Определение \ (\ PageIndex {3} \): Форма «ряд-эшелон»

Расширенная матрица находится в -строчной форме , если

  1. Все ненулевые строки находятся над любыми строками с нулями.
  2. Каждая ведущая запись строки находится в столбце справа от ведущих записей любой строки над ней.
  3. Каждая ведущая запись строки равна \ (1 \).

Мы также рассматриваем другую сокращенную форму расширенной матрицы, которая имеет еще одно условие.

Определение \ (\ PageIndex {4} \): сокращенная форма строки-эшелона

Расширенная матрица находится в сокращенной форме строки-эшелона , если

  1. Все ненулевые строки находятся над любыми строками с нулями.
  2. Каждая ведущая запись строки находится в столбце справа от ведущих записей любых строк над ней.
  3. Каждая ведущая запись строки равна \ (1 \).
  4. Все записи в столбце выше и ниже ведущей записи равны нулю.

Обратите внимание, что первые три условия в матрице сокращенной формы «строка-эшелон» такие же, как и для формы «строка-эшелон».

Следовательно, каждая сокращенная матрица формы «строка-эшелон» также находится в форме «строка-эшелон». Обратное не обязательно верно; мы не можем предположить, что каждая матрица в форме эшелона строк также находится в сокращенной форме эшелона строк.Однако часто бывает, что формы «ряд строк» ​​достаточно, чтобы предоставить информацию о решении системы.

Следующие ниже примеры описывают матрицы в этих различных формах. В качестве упражнения найдите время, чтобы тщательно убедиться, что они находятся в указанной форме.

Пример \ (\ PageIndex {5} \): отсутствует в форме «Ряд-эшелон»

Следующие расширенные матрицы не имеют формы «строка-эшелон» (и, следовательно, также не имеют сокращенной формы «строка-эшелон»).

\ [\ left [\ begin {array} {rrr | r} 0 & 0 & 0 & 0 \\ 1 & 2 & 3 & 3 \\ 0 & 1 & 0 & 2 \\ 0 & 0 & 0 & 1 \\ 0 & 0 & 0 & 0 \ end {array} \ right], \ left [\ begin {array} {rr | r} 1 & 2 & 3 \\ 2 & 4 & -6 \\ 4 & 0 & 7 \ end {array} \ right], \ left [\ begin {array} {rrr | r} 0 & 2 & 3 & 3 \\ 1 & 5 & 0 & 2 \\ 7 & 5 & 0 & 1 \\ 0 & 0 & 1 & 0 \ end {array} \ right] \]

Пример \ (\ PageIndex {6} \): матрицы в форме строки-эшелон

Следующие расширенные матрицы представлены в форме эшелонов строк, но не в сокращенной форме эшелонов строк.\ [\ left [\ begin {array} {rrrrr | r} 1 & 0 & 6 & 5 & 8 & 2 \\ 0 & 0 & 1 & 2 & 7 & 3 \\ 0 & 0 & 0 & 0 & 1 & 1 \\ 0 & 0 & 0 & 0 & 0 & 0 \ end {array} \ right], \ left [\ begin {array} {rrr | r} 1 & 3 & 5 & 4 \\ 0 & 1 & 0 & 7 \\ 0 & 0 & 1 & 0 \\ 0 & 0 & 0 & 1 \\ 0 & 0 & 0 & 0 \ end {array} \ right], \ left [\ begin {array} {rrr | r} 1 & 0 & 6 & 0 \\ 0 & 1 & 4 & 0 \\ 0 & 0 & 1 & 0 \\ 0 & 0 & 0 & 0 \ end {array} \ right] \]

Обратите внимание, что мы можем применить дополнительные операции со строками к этим матрицам, чтобы преобразовать их в сокращенную форму строки-эшелона.Найдите время, чтобы попробовать это самостоятельно. Рассмотрим следующие матрицы, приведенные в сокращенном виде по строкам.

Пример \ (\ PageIndex {7} \): матрицы в сокращенной форме строки-эшелон

Следующие расширенные матрицы представлены в сокращенной форме строки-эшелона. \ [\ left [\ begin {array} {rrrrr | r} 1 & 0 & 0 & 5 & 0 & 0 \\ 0 & 0 & 1 & 2 & 0 & 0 \\ 0 & 0 & 0 & 0 & 1 & 1 \\ 0 & 0 & 0 & 0 & 0 & 0 \ end {array} \ right], \ left [\ begin {array} {rrr | r} 1 & 0 & 0 & 0 & 0 \\ 0 & 1 & 0 & 0 \\ 0 & 0 & 1 & 0 \\ 0 & 0 & 0 & 1 \\ 0 & 0 & 0 & 0 \ end {array} \ right], \ left [\ begin {array} {rrr | r} 1 & 0 & 0 & 4 \\ 0 & 1 & 0 & 3 \\ 0 & 0 & 1 & 2 \ end {array} \ right] \]

Одним из способов использования формы строки-эшелона матрицы является идентификация опорных позиций и опорных столбцов матрицы.

Определение \ (\ PageIndex {8} \): положение поворота и столбец поворота

Позиция поворота в матрице — это положение ведущей записи в матричной форме «строка-эшелон».

Сводный столбец — это столбец, который содержит сводную позицию.

Например, рассмотрим следующее.

Пример \ (\ PageIndex {9} \): позиция поворота

Пусть \ [A = \ left [\ begin {array} {rrr | r} 1 & 2 & 3 & 4 \\ 3 & 2 & 1 & 6 \\ 4 & 4 & 4 & 10 \ end {array} \ справа] \] Где находятся опорные позиции и опорные столбцы расширенной матрицы \ (A \)?

Решение

Строка-эшелон этой матрицы: \ [\ left [\ begin {array} {rrr | r} 1 & 2 & 3 & 4 \\ 0 & 1 & 2 & \ frac {3} {2} \\ 0 & 0 & 0 & 0 \ end {array} \ right] \]

Это все, что нам нужно в этом примере, но обратите внимание, что эта матрица не в сокращенной форме строки-эшелона.

Для того, чтобы идентифицировать опорные позиции в исходной матрице, мы ищем ведущие элементы в форме строки-эшелона матрицы. Здесь запись в первой строке и первом столбце, а также запись во второй строке и втором столбце являются ведущими записями. Следовательно, эти местоположения являются опорными. Мы идентифицируем опорные позиции в исходной матрице, как показано ниже: \ [\ left [\ begin {array} {rrr | r} \ fbox {1} & 2 & 3 & 4 \\ 3 & \ fbox {2} & 1 & 6 \\ 4 & 4 & 4 & 10 \ end {array} \ right] \] Таким образом, сводные столбцы в матрице — это первые два столбца.

Ниже приведен алгоритм преобразования матрицы в форму строки-эшелон и сокращенную форму строки-эшелон. Вы можете использовать этот алгоритм для переноса указанной выше матрицы в форму строки-эшелон или сокращенную форму строки-эшелон самостоятельно для практики.

Алгоритм \ (\ PageIndex {10} \): алгоритм сокращенной строки-эшелона

Этот алгоритм обеспечивает метод использования строковых операций для приведения матрицы к ее сокращенной форме «строка-эшелон». Начнем с матрицы в первоначальном виде.

  1. Найдите первый ненулевой столбец слева. Это первый поворотный столбец, и позиция вверху этого столбца является первым поворотным столбцом. При необходимости поменяйте строки, чтобы поместить ненулевое число в первую позицию поворота.
  2. Используйте операции со строками, чтобы сделать записи ниже первой позиции поворота (в первом столбце поворота) равными нулю.
  3. Игнорируя строку, содержащую первую позицию поворота, повторите шаги 1 и 2 с оставшимися строками.Повторяйте процесс до тех пор, пока не останется строк, которые нужно изменить.
  4. Разделите каждую ненулевую строку на значение ведущей записи, чтобы ведущая запись стала \ (1 \). В этом случае матрица будет в виде эшелона строк.

    На следующем шаге матрица будет переведена из формы «строка-эшелон» в сокращенную форму «строка-эшелон».

  5. Двигаясь справа налево, используйте операции со строками для создания нулей в записях сводных столбцов, которые находятся над их положениями. Результатом будет матрица в уменьшенном виде эшелона строк.

Чаще всего мы будем применять этот алгоритм к расширенной матрице, чтобы найти решение системы линейных уравнений. Тем не менее, мы можем использовать этот алгоритм для вычисления сокращенной строковой формы любой матрицы, которая может быть полезна в других приложениях.

Рассмотрим следующий пример алгоритма [algo: rrefalgorithm].

Пример \ (\ PageIndex {11} \) Поиск формы «строка-эшелон» и
сокращенной формы «строка-эшелон» матрицы

Пусть \ [A = \ left [\ begin {array} {rrr} 0 & -5 & -4 \\ 1 & 4 & 3 \\ 5 & 10 & 7 \ end {array} \ right] \] Найдите строчно-эшелонированная форма \ (A \).Затем завершайте процесс до тех пор, пока \ (A \) не перейдет в сокращенную форму звеньев.

Решение

При работе с этим примером мы будем использовать шаги, описанные в алгоритме [algo: rrefalgorithm].

  1. Первый сводный столбец — это первый столбец матрицы, так как это первый ненулевой столбец слева. Следовательно, первая точка поворота — это позиция в первой строке и первом столбце. Переключите первые две строки, чтобы получить ненулевую запись в первой опорной позиции, выделенной в поле ниже.\ [\ left [\ begin {array} {rrr} \ fbox {1} & 4 & 3 \\ 0 & -5 & -4 \\ 5 & 10 & 7 \ end {array} \ right] \]
  2. Шаг второй включает в себя создание нулей в записях под первой опорной позицией. Первая запись второй строки уже равна нулю. Все, что нам нужно сделать, это вычесть первую строку из третьей в \ (5 \) раз. В результате получается матрица \ [\ left [\ begin {array} {rrr} 1 & 4 & 3 \\ 0 & -5 & -4 \\ 0 & 10 & 8 \ end {array} \ right] \]
  3. Теперь игнорируйте верхнюю строку.Примените шаги \ (1 \) и \ (2 \) к меньшей матрице \ [\ left [\ begin {array} {rr} -5 & -4 \\ 10 & 8 \ end {array} \ right] \] В этой матрице первый столбец является сводным столбцом, а \ (- 5 \) находится в первой сводной позиции. Следовательно, нам нужно создать ноль под ним. Для этого прибавьте \ (2 \) раз первую строку (этой матрицы) ко второй. В результате получается матрица \ [\ left [\ begin {array} {rr} -5 & -4 \\ 0 & 0 \ end {array} \ right] \] Наша исходная матрица теперь выглядит как \ [\ left [\ begin {array} {rrr} 1 & 4 & 3 \\ 0 & -5 & -4 \\ 0 & 0 & 0 \ end {array} \ right] \] Мы видим, что больше нет строк для изменения.
  4. Теперь нам нужно создать ведущие \ (1 \) в каждой строке. В первой строке уже есть ведущий \ (1 \), поэтому здесь не нужно работать. Разделите вторую строку на \ (- 5 \), чтобы создать ведущую \ (1 \). В результате получается матрица \ [\ left [\ begin {array} {rrr} 1 & 4 & 3 \\ 0 & 1 & \ frac {4} {5} \\ 0 & 0 & 0 \ end {array} \ right ] \] Эта матрица теперь имеет строковую форму.
  5. Теперь создайте нули в записях над позициями поворота в каждом столбце, чтобы полностью перенести эту матрицу в сокращенную форму строки-эшелона.Обратите внимание, что в третьем столбце нет точки поворота, поэтому нам не нужно создавать какие-либо нули в этом столбце! Столбец, в котором нам нужно создать нули, — второй. Для этого вычтите \ (4 \) раз вторую строку из первой. В результате получается матрица \ [\ left [\ begin {array} {rrr} 1 & 0 & — \ frac {1} {5} \\ 0 & 1 & \ frac {4} {5} \\ 0 & 0 & 0 \ end {array} \ right] \]

Эта матрица теперь имеет сокращенную форму строки-эшелона.

Приведенный выше алгоритм дает вам простой способ получить форму строки-эшелон и сокращенную форму строки-эшелон матрицы.Основная идея состоит в том, чтобы выполнять операции со строками таким образом, чтобы в итоге получить матрицу в форме эшелона строк или сокращенной форме эшелона строк. Этот процесс важен, потому что полученная матрица позволит вам содержательно описать решения соответствующей линейной системы уравнений.

В следующем примере мы рассмотрим, как решить систему уравнений, используя соответствующую расширенную матрицу.

Пример \ (\ PageIndex {12} \): поиск решения для системы

Дайте полное решение следующей системы уравнений \ [\ begin {array} {c} 2x + 4y-3z = -1 \\ 5x + 10y-7z = -2 \\ 3x + 6y + 5z = 9 \ end {array} \]

Решение

Расширенная матрица для этой системы: \ [\ left [\ begin {array} {rrr | r} 2 & 4 & -3 & -1 \\ 5 & 10 & -7 & -2 \\ 3 & 6 & 5 & 9 \ end {array} \ right] \]

Чтобы найти решение этой системы, мы хотим перенести расширенную матрицу на.Мы сделаем это с помощью алгоритма [algo: rrefalgorithm]. Обратите внимание, что первый столбец отличен от нуля, поэтому это наш первый сводный столбец. Первая запись в первой строке, \ (2 \), является первой ведущей записью и находится в первой позиции поворота. Мы будем использовать операции со строками для создания нулей в записях под \ (2 \). Сначала замените вторую строку на \ (- 5 \), умноженную на первую строку, плюс \ (2 \), умноженную на вторую строку. Это дает \ [\ left [\ begin {array} {rrr | r} 2 & 4 & -3 & -1 \\ 0 & 0 & 1 & 1 \\ 3 & 6 & 5 & 9 \ end {array} \ right] \] Теперь замените третью строку на \ (- 3 \), умноженную на первую строку, плюс на \ (2 \), умноженную на третью строку.Это дает \ [\ left [\ begin {array} {rrr | r} 2 & 4 & -3 & -1 \\ 0 & 0 & 1 & 1 \\ 0 & 0 & 1 & 21 \ end {array} \ right] \] Теперь записи в первом столбце под опорной позицией нулевые. Теперь мы ищем второй сводный столбец, которым в данном случае является третий столбец. Здесь \ (1 \) во второй строке и третьем столбце находится в опорной позиции. Нам нужно выполнить только одну строковую операцию, чтобы создать ноль ниже \ (1 \).

Если взять \ (- 1 \) раз вторую строку и добавить ее к третьей строке, получим \ [\ left [\ begin {array} {rrr | r} 2 & 4 & -3 & -1 \\ 0 & 0 & 1 & 1 \\ 0 & 0 & 0 & 20 \ end {array} \ right] \]

Мы могли бы перейти к алгоритму преобразования этой матрицы в форму строки-эшелон или сокращенную форму строки-эшелона.Однако помните, что мы ищем решения системы уравнений. Еще раз взгляните на третью строку матрицы. Обратите внимание, что это соответствует уравнению \ [0x + 0y + 0z = 20 \]. У этого уравнения нет решения, потому что для всех \ (x, y, z \) левая часть будет равна \ (0 \) и \ (0 \ neq 20. \) Это показывает, что у данной системы уравнений нет решения. Другими словами, эта система непоследовательна.

Ниже приводится еще один пример того, как найти решение системы уравнений путем преобразования соответствующей расширенной матрицы в сокращенную форму строки-эшелона.

Пример \ (\ PageIndex {13} \): бесконечный набор решений

Дайте полное решение системы уравнений \ [\ begin {array} {c} 3x-y-5z = 9 \\ y-10z = 0 \\ -2x + y = -6 \ end {array} \]

Решение

Расширенная матрица этой системы: \ [\ left [\ begin {array} {rrr | r} 3 & -1 & -5 & 9 \\ 0 & 1 & -10 & 0 \\ -2 & 1 & 0 & -6 \ end {array} \ right] \] Для того, чтобы найти решение этой системы, мы приведем расширенную матрицу к сокращенной форме эшелона строки, используя алгоритм [algo: rrefalgorithm].Первый столбец — это первый сводный столбец. Мы хотим использовать операции со строками для создания нулей под первой записью в этом столбце, которая находится в первой позиции поворота. Замените третью строку на \ (2 \), умноженную на первую, добавленную к \ (3 \), умноженную на третью. Это дает

\ [\ left [\ begin {array} {rrr | r} 3 & -1 & -5 & 9 \\ 0 & 1 & -10 & 0 \\ 0 & 1 & -10 & 0 \ end {array} \ right] \]

Теперь мы создали нули под \ (3 \) в первом столбце, поэтому мы переходим ко второму столбцу поворота (который является вторым столбцом) и повторяем процедуру.Возьмите \ (- 1 \) раз второй ряд и прибавьте к третьему ряду. \ [\ left [\ begin {array} {rrr | r} 3 & -1 & -5 & 9 \\ 0 & 1 & -10 & 0 \\ 0 & 0 & 0 & 0 & 0 \ end {array} \ right ] \] Запись под точкой поворота во втором столбце теперь равна нулю. Обратите внимание, что у нас больше нет сводных столбцов, потому что у нас есть только две ведущие записи.

На этом этапе мы также хотим, чтобы ведущие записи были равны единице. Для этого разделите первую строку на \ (3 \). \ [\ left [\ begin {array} {rrr | r} 1 & — \ frac {1} {3} & — \ frac {5} {3} & 3 \\ 0 & 1 & -10 & 0 \\ 0 & 0 & 0 & 0 \ end {array} \ right] \]

Эта матрица теперь имеет строковую форму.

Давайте продолжим работу со строками, пока матрица не перейдет в сокращенную форму строки-эшелона. Это включает в себя создание нулей над опорными позициями в каждом сводном столбце. Для этого требуется только один шаг — добавить \ (\ frac {1} {3} \) раз вторую строку к первой строке. \ [\ left [\ begin {array} {rrr | r} 1 & 0 & -5 & 3 \\ 0 & 1 & -10 & 0 \\ 0 & 0 & 0 & 0 & 0 \ end {array} \ right] \]

Это сокращенная форма строки, которую вы должны проверить с помощью Definition [def: rref].Уравнения, соответствующие этой сокращенной форме строки-эшелона, следующие: \ [\ begin {array} {c} x — 5z = 3 \\ y — 10z = 0 \ end {array} \] или \ [\ begin {array} {c } x = 3 + 5z \\ y = 10z \ end {array} \]

Обратите внимание, что \ (z \) не ограничивается никаким уравнением. Фактически, \ (z \) может равняться любому числу. Например, мы можем положить \ (z = t \), где мы можем выбрать \ (t \) как любое число. В этом контексте \ (t \) называется параметром . Следовательно, набор решений этой системы равен \ [\ begin {array} {c} x = 3 + 5t \\ y = 10t \\ z = t \ end {array} \], где \ (t \) произвольно.Система имеет бесконечный набор решений, которые даются этими уравнениями. Для любого значения \ (t \), которое мы выбираем, \ (x, y, \) и \ (z \) будут определяться приведенными выше уравнениями. Например, если мы выберем \ (t = 4 \), то соответствующее решение будет \ [\ begin {array} {c} x = 3 + 5 (4) = 23 \\ y = 10 (4) = 40 \ \ z = 4 \ end {array} \]

В примере [exa: infinitesetofsoln] решение включает один параметр. Может случиться так, что решение системы включает более одного параметра, как показано в следующем примере.

Пример \ (\ PageIndex {1} \): двухпараметрический набор решений

Найдите решение системы \ [\ begin {array} {c} x + 2y-z + w = ​​3 \\ x + y-z + w = ​​1 \\ x + 3y-z + w = ​​5 \ end {array} \]

Решение

Расширенная матрица: \ [\ left [\ begin {array} {rrrr | r} 1 & 2 & -1 & 1 & 3 \\ 1 & 1 & -1 & 1 & 1 & 1 \\ 1 & 3 & -1 & 1 & 5 \ end {array} \ right] \] Мы хотим привести эту матрицу к строковой форме. Здесь мы опишем используемые строковые операции.Однако убедитесь, что вы понимаете шаги с точки зрения алгоритма [algo: rrefalgorithm].

Возьмите \ (- 1 \) раз первый ряд и прибавьте ко второму. Затем возьмите \ (- 1 \) раз первый ряд и прибавьте к третьему. Это дает \ [\ left [\ begin {array} {rrrr | r} 1 & 2 & -1 & 1 & 3 \\ 0 & -1 & 0 & 0 & -2 \\ 0 & 1 & 0 & 0 & 2 \ end {array} \ right] \]

Теперь добавьте вторую строку к третьей и разделите вторую строку на \ (- 1 \). \ [\ left [\ begin {array} {rrrr | r} 1 & 2 & -1 & 1 & 3 \\ 0 & 1 & 0 & 0 & 2 \\ 0 & 0 & 0 & 0 & 0 \ end { массив} \ right] \ label {twoparameters1} \]

Эта матрица находится в виде эшелона строк, и мы можем видеть, что \ (x \) и \ (y \) соответствуют столбцам сводной таблицы, а \ (z \) и \ (w \) — нет.Поэтому мы присвоим параметры переменным \ (z \) и \ (w \). Присвойте параметр \ (s \) к \ (z \) и параметр \ (t \) к \ (w. \). Тогда первая строка дает уравнение \ (x + 2y-s + t = 3 \), а вторая строка дает уравнение \ (y = 2 \). Поскольку \ (y = 2 \), первое уравнение становится \ (x + 4-s + t = 3 \), показывая, что решение задается \ [\ begin {array} {c} x = -1 + st \ \ y = 2 \\ z = s \\ w = t \ end {array} \] Это решение принято записывать в виде \ [\ left [\ begin {array} {c} x \\ y \\ z \\ w \ end {массив} \ right] = \ left [\ begin {array} {c} -1 + st \\ 2 \\ s \\ t \ end {array} \ right] \ label {twoparameters2} \]

В этом примере показана система уравнений с бесконечным множеством решений, которое зависит от двух параметров.Это может быть менее запутанным в случае бесконечного набора решений, чтобы сначала поместить расширенную матрицу в сокращенную форму строки-эшелон, а не просто форму строки-эшелон, прежде чем пытаться записать описание решения.

В описанных выше шагах это означает, что мы не останавливаемся на форме «строка-эшелон» в уравнении [twoparameters1]. Вместо этого мы сначала помещаем его в сокращенную форму эшелона строк следующим образом. \ [\ left [\ begin {array} {rrrr | r} 1 & 0 & -1 & 1 & -1 \\ 0 & 1 & 0 & 0 & 2 \\ 0 & 0 & 0 & 0 & 0 \ end {array} \ right] \] Тогда решением будет \ (y = 2 \) из второй строки и \ (x = -1 + zw \) из первой.Таким образом, если принять \ (z = s \) и \ (w = t, \), решение задается [двумя параметрами2].

Здесь вы можете видеть, что есть два пути к правильному ответу, и оба дают один и тот же ответ. Следовательно, можно использовать любой подход. Процесс, который мы впервые использовали в вышеупомянутом решении, называется Исключение Гаусса Этот процесс включает перевод матрицы в строковую форму, обратное преобразование в уравнения и использование обратной подстановки для поиска решения. Когда вы выполняете операции со строками до тех пор, пока не получите сокращенную форму ряда строк, процесс называется Исключение Гаусса-Джордана .

Мы нашли решения для систем уравнений без решения и с бесконечным множеством решений, с одним параметром, а также с двумя параметрами. Вспомните три типа наборов решений, которые мы обсуждали в предыдущем разделе; нет решения, одно решение и бесконечно много решений. Каждый из этих типов решений можно идентифицировать по графику системы.

Добавить комментарий

Ваш адрес email не будет опубликован. Обязательные поля помечены *